Genetics Board Stat Pearls Flashcards
Capillary Malformations/Port Wine Stains
Present at birth
Can have underlying soft tissue or bone hypertrophy (one leg can be bigger than the other)
Can darken, become thick, raised, nodular
GNAQ gene
Familial Venous Malformation Syndrome
TEK gene
Proteus Syndrome
ATK1
Capillary Malformation Artiovenous Malformation Syndrome
RASA1
Werner Syndrome
Normal development until adolescence and then notice signs of accelerated aging
Skin ulcers, cataracts, graying, hypogonadism
50% of malignancies are soft tissue sarcomas, schwannoma, rhabdomyosarcoma, etc
10% have malignancy
Life expectancy is around 40-50 years
PKAN
Pantothenate kinase associated neurodegeneration
Neurocanthocytosis syndrome caused by PKAN2 gene (AR)
Presents in 1st decade, rapid progression (typical)
2nd/3rd decade, slow progression (atypical)
EPS dysfunction, loss of ambulation, eye of the tiger sign on MRI with hypodensities of the globus pallidus
Iron chelators and dopominergic drugs most helpful
Klippel Trenauney
Venous malformations, port wine stains, and/or angiokerotoma circumscriptum
Usually affects single extremity, the leg being the most common for bony and tissue hypertrophy
CLL Prognostic Factors
17p deletion - poor prognosis and advise stem cell transplant especially if patient is young
Trisomy 12 - intermediate prognosis
13q deletion - favorable prognosis
Lynch Syndrome
MSH1/2/6, PMS2
Colorectal adenomas
Endometrial cancer
Less common: gastric, ovarian, urothelial cancer -> usually after endometrial
Apoprotein CII Deficiency
Apoprotein CII deficiency usually involves recurrent acute pancreatitis, demonstrated by increased blood concentration of amylase and lipase.
his disorder is typically diagnosed later in childhood. Chylomicrons and very-low-density lipoprotein (VLDL) levels are elevated in addition to a significant elevation of triglycerides.
Globoid leukodystrophy
Globoid leukodystrophy or Krabbe disease shows autosomal recessive inheritance and has reduced galactosylceramide beta-galactosidase activity.
There are severe myelin loss and white matter globoid bodies. Due to galactocerebrosidase deficiency, psychosine cant is degraded. As a result, there is an accumulation of psychosine, which causes pathologic effects in the central and peripheral nervous system (globoid cell formation and decreased myelin) due to its toxic nature.
There is an early and a late form, both with seizures, psychomotor arrest, and vision loss.
Late infantile-onset Krabbe disease presents between 13 months and 36 months with irritability, abnormal gait, or vision difficulties. Motor milestones are typically reached within the typical age range. As the disease progresses, visual difficulty, apneic episodes, temperature instability, and seizures are more likely. The median age of death is six years.
Polyglandular Autoimmune Syndrome
Polyglandular autoimmune syndrome type 2 is diagnosed by the occurrence of at least 2 out of 3 manifestations, including Addison disease, autoimmune thyroid disease-causing (Graves disease or hypothyroidism), and T1DM.
Other endocrine and non-endocrine manifestations of PAS-2 include primary hypogonadism, myasthenia gravis, celiac disease, alopecia, vitiligo, pernicious anemia, idiopathic heart block, Stiff-man syndrome, Parkinson disease, IgA deficiency, serositis, dermatitis herpetiformis, idiopathic thrombocytopenia, and hypophysitis.
Mutation in HLA-DR3 and HLA-DR4 genes is commonly seen in polyglandular autoimmune syndrome type 2. A mutation in the CD25-interleukin-2 receptor gene, CTLA-4 gene, or protein tyrosine phosphatase can also cause autoimmune polyglandular syndrome type 2.
Cancer Prognosis Factors
Complex karyotype with greater than 5 abnormalities - most important independent of TP53 mutation status
In cases of 3-4 chromosomal abnormalities, prognosis is dependent on TP53 status
Syndrome Associated with Double Aortic Arches
22q11 deletion
DAA presents in early infancy with stridor, dyspnea, FTT, and choking episodes with feeding
Syndrome Associated with Pulmonary Valvular Stenosis and ASDs
Noonan
Fanconi Anemia
ID, microcephaly, renal complications, short radii, hypoplastic thumbs, and anemia; pancytopenia (all three cell lines)
Shortness of breath, chest pain, dizziness, fatiguability are common manifestations of anemia. History of epistaxis, petechiae, unstoppable bleeding from the wound site is common due to thrombocytopenia, and chances of recurrent infections increase with the severity of leukopenia which presents with fever and flu-like illness. History low birth weight is present in some cases and weight loss is important to rule out those cases complicated by cancer. Family history and marriage history are important particularly where the prevalence of the consanguinity marriage system is high.
Defect in homologous recombination of double-stranded DNA is the main mechanism of the pathogenesis of Fanconi anemia.
Various FA proteins maintain genomic stability through DNA interstrand crosslinks (ICLs) repair. ICLs prevent DNA strand separation and maintain DNA integrity. Genetic defects in DNA repair FA pathway so that cells cannot properly repair especially detrimental types of DNA damage.
HSCT treatment if ANC <500, Hgb <8, plts <30,000
Hypokalemic Periodic Paralysis
Hypokalemic periodic paralysis has been linked to a mutation at chromosome 1q32 of the calcium channel and with a mutation at 17q23.1-q25.3 of a sodium channel.
The type 1 hypokalemic periodic paralysis is the most common familial form. There is a mutation in the dihydropyridine-sensitive, skeletal muscle calcium channel gene.
There is autosomal dominant inheritance, but may patients have sporadic disease.
Adolescent onset is most common with precipitating factors, including stress, exercise, carbohydrates, sodium intake, sleep, and alcohol.
12 year old with ataxia, dystonia, and supranuclear gaze palsy
Niemann-Pick disease type C
Niemann-Pick disease type C is caused y a deficiency of enzyme sphingomyelinase, causing a buildup of cholesterol inside cells.
For suspected type C disease, the enzyme activity is measured by taking a skin biopsy and staining it with filipin.
Type A and B mostly present in the first few months of life. Type A is very severe, and the affected children do not live past four years of age. Type B is less severe and has minimal neurological symptoms.
Carney Complex
The patient described above has Carney complex (CNC), given the diagnosis of acromegaly, cardiac myxoma, skin lesions, and thyroid nodule. CNC gene 1 is a germline mutation in a regulatory subunit 1A of protein kinase A (PRKAR1A) located at 17q22-24 observed in about two-thirds of Carney complex patients. Some patients may experience other endocrine problems such as hirsutism due to adrenal tumors.
The syndrome has an autosomal dominant inheritance. It often presents with endocrinopathy, patch skin pigmentation, and neuroendocrine tumors. Primary pigmented adrenocortical disease (PPNAD) is commonly seen in these patients (not primary macronodular adrenal hyperplasia). It is characterized by the small-pigmented nodules less than 10 mm in their greatest diameter most often surrounded by the atrophic cortex.
Myxomas often occur in the left atrium, breast, and skin. The cardiac exam will reveal a loud S1 and a diastolic rumbling murmur. The plopping of the tumor from the left atrium to the left ventricle may also be heard.
Twenty percent to 50% of Carney patients have at least one of the testicular tumors, including large-cell calcifying Sertoli cell tumor (LCCST), nodular adrenocortical rests, and Leydig cell tumors.
Which collagen type causes fatal OI?
Type II
AR or dominant negative
CHARGE
CHARGE syndrome is characterized by coloboma, heart defects, choanal atresia (bilateral), genital abnormalities, and ear abnormalities.
MSUD
Classic maple-syrup urine disease (MSUD) occurs due to reduced branched-chain ketoacid dehydrogenase (BCKAD) activity of 0% to 2% of normal. BCKAD is located within the inner mitochondrial membrane of various tissues such as skeletal muscle, liver, kidney, and the brain.
Clinical constellation of vomiting, lethargy, seizures, abnormal neurological findings and elevated valine, leucine, allo-isoleucine, and isoleucine.
If left untreated, neonates develop irritability, poor feeding, lethargy, opisthotonus, dystonias, central respiratory failure, coma, maple syrup odor of cerumen, neurological complications, and stereotypical movements such as fencing and bicycling.
Branched Chain AAs
The branched-chain amino acids (BCAA) are essential amino acids with hydrophobic side chains and are found in protein-rich food. Their catabolism is necessary to maintain various physiologic functions such as protein synthesis, gluconeogenesis, fatty acid synthesis, cholesterol synthesis, and cellular signaling. In the brain, BCKAD helps metabolize BCAA to facilitate cerebral GABA and glutamate synthesis. The liver and kidney are responsible for the catabolism of 10% to 15% of BCAA. However, unlike most amino acids, the majority of BCAA transamination and oxidation takes place in the skeletal muscle.
McLeod Syndrome
McLeod syndrome is a neuroacanthocytosis syndrome that results from an X-linked recessive inherited mutation in the XK gene
XK protein is expressed in blood, brain, and muscle cells but appears to differ in function among cell types. Loss of XK function leads to abnormal RBC membrane shape due to loss of disulfide bonding with Kell glycoprotein. Neuronal and cardiac involvement is not well understood.
Patients diagnosed with McLeod syndrome are at significantly increased risk of developing congestive or dilated cardiomyopathy and subsequent cardiac arrhythmias (e.g., atrial fibrillation) that can lead to premature death.
ECG or Holter monitor every 2-3 years with appropriate cardiology evaluation and follow-up to prevent morbidity and premature mortality.
Can be a differential when suspecting Huntington disease. It causes atrophy of the caudate nucleus and putamen on CT brain and transfusion reactions. Huntington has significant striatal, putamen, and caudate volume loss, but mostly of the striatum.
Crouzon Syndrome
Crouzon syndrome is inherited in an autosomal dominant pattern and is caused by a mutation in the fibroblast growth factor receptor (FGFR)-2 and -3 on chromosome 10.
Triad of skull deformities, facial anomalies, and proptosis.
Crouzon syndrome is a fairly rare entity and is estimated to occur in 1 in 60,000 newborns; however, it is the second most common craniosynostosis syndrome behind only the more recently described Muenke syndrome.
Osteopetrosis
“Rugger Jersey spine”
Due to failure of osteoclasts
Anemia due to bone marrow sclerosis
AD- less severe/asymptomatic until pathologic fracture occurs
AR- severe, multiple pathologic fractures, earlier onset
WAGR
About two-thirds of patients with aniridia follow an autosomal dominant pattern.
For de novo variant:
Sporadic cases account for one-third of patients. A rare autosomal recessive variant (Gillespie syndrome) is associated with aniridia, cerebellar ataxia, and intellectual disability.
The implicated gene abnormality is 11p13. Sporadic aniridia may be associated with WAGR syndrome (Wilm tumor, aniridia, genitourinary anomalies, and mental retardation).
In sporadic congenital aniridia, the Wilm tumor must be ruled out by ultrasonography of the abdomen.
Holt Oram
“Heart hand”
examination reveals the presence of hand and upper limb abnormalities (triphalangeal thumb and radial dysplasia). An echocardiogram, if done, would confirm the presence of atrial secundum typical of the syndrome.
Holt-Oram syndrome is an autosomal dominant disorder with complete penetrance. The underlying defect is found in the long arm of chromosome 12 (12q2), which contains the genes TBX5 and TBX3. Mutation of these genes gives an embryologic prevalence for ASD, VSD, and left-sided malformation.
Radial aplasia can also be seen in syndromes like Fanconi anemia, thrombocytopenia absent radius (TAR) syndrome, and VACTERL. Fanconi anemia and TAR syndrome show autosomal recessive inheritance. VACTERL has a sporadic inheritance.
Ataxia Telangiectasia
Progressive cerebellar ataxia, oculomotor apraxia, oculocutaneous telangiectasia, etc.
Degeneration of purkinje and granular cells in the cerebellum
25-30% develop neoplasia, most commonly leukemia and lymphoma early in life
Later in life - breast, ovarian, melanoma, gastric, liver tumors. Associated with severe radio sensitivity so avoiding XR and gamma rays. UV is ok.
Lipoprotein Lipase Deficiency
LPL deficiency is the commonest cause of familial chylomicronemia in children.
It is an autosomal recessive disorder and due to homozygous or compound heterozygous mutations of LPL.
It can result in severe chylomicronemia which increases the risk for pancreatitis.
It invariably presents in infancy or childhood.
A 6-month-old boy presents with a cleft palate and micrognathia. A careful clinical examination shows a short neck, cervical webs, an abnormal curvature of the spine, and excessive hair on the entire body. He also demonstrates an abnormal gaze on the eye exam. Upon further inquiry, his parents report frequent episodes of gastroesophageal reflux. Genetic testing confirms the suspicion for a mutation on a gene located in the short arm (p) of chromosome 5 at position 13.2. Which of the following is most likely to be present in this infant?
Children with Cornelia de Lange syndrome often develop intrauterine growth restriction, resulting in low birth weight (less than 2.2 kg). Children that are small for gestational age will have prolonged difficulty gaining weight.
Gastrointestinal problems are a key manifestation of patients with Cornelia de Lange syndrome. Gastrointestinal problems may include poor appetite, feeding problems, gastrointestinal reflux disease (GERD), vomiting, diarrhea, and constipation.
Special diets that incorporate supplemental formulas can provide the nutrition necessary to overcome feeding and weight gain issues
MRI Findings in Huntington Disease
Striatum atrophy (caudate and putamen)
Jerking movements, irritability, and dementia
Acrodysostosis Type 1
PTH resistance is a common feature of pseudohypoparathyroidism and acrodysostosis with hormonal resistance (acrodysostosis type 1).
Acrodysostosis type 1 occurs due to an inactivating mutation in PRKR1A.
Hypocalcemia, hyperphosphatemia, and elevated PTH suggest PTH resistance. PTH resistance should be suspected even when serum calcium is normal.
If evidence of PTH resistance is present, evaluation for other hormonal resistance (serum thyroid-stimulating hormone and serum-free thyroxine) should be conducted. Hormonal resistance is not a feature of acrodysostosis type 2 (PDE4D mutation).
A 12-day-old infant is brought to the emergency department with complaints of lethargy and poor feeding. Further questioning reveals that these symptoms gradually worsened from the 8th day of life. The infant has also developed episodes of irregular breathing since yesterday night. A dinitrophenylhydrazine test reveals opaque urine and a strong yellow precipitate. Further clinical evaluation reveals a sweet odor from the ears of the infant and elevated urinary branched-chain ketoacids. Plasma amino acid analysis is also significant for elevated levels of branched-chain amino acids. A defect in which of the following biochemical processes is responsible for the infants’ condition?
Decarboxylation of branched-chain alpha-keto acids
Maple syrup urine disease occurs due to a pathogenic defect in any subunit of the branched-chain ketoacid dehydrogenase enzyme complex. Other diagnostic tests include a dinitrophenylhydrazine (DNPH) test. DNPH test can be used as a screening modality in newborns greater than 48-72 hours of age. DNPH reagent and urine are mixed in equal volumes. The mixed sample is observed for 10 minutes for precipitation and color change. Clear urine with no precipitate indicates a score of zero. Whereas, a yellow-white precipitate with opaque urine indicates a score of 4. In our patient, a positive test result is noted.
Branched-chain amino acid transaminase converts leucine, isoleucine, and valine into their respective alpha-ketoacids. Branched-chain ketoacid dehydrogenase (BCKAD) is the second enzyme in the pathway of branched-chain amino acid metabolism. It is responsible for the oxidative decarboxylation of their respective alpha-ketoacids. A defect in BCKAD can result in elevations of alpha-ketoacids, which are responsible for the clinicopathological manifestations of maple syrup urine disease.
There are five subtypes of MSUD. These include the classic, intermediate, intermittent, thiamine responsive, and E3 deficient subtypes. They are clinically distinguished based on their clinical presentation, age of onset, and residual BCKAD enzyme activity.
The mainstay of treatment is a dietary restriction of branched-chain amino acids and regular monitoring of blood chemistry.
Lynch Syndrome Cancers
Lynch syndrome is associated with an increased risk of upper tract urothelial cancer. They tend to have a higher involvement of the ureters, are more likely to be found in females, and there is a possible predisposition to bilaterality.
TCC of the upper tracts can be found in up to 28% of patients with Lynch syndrome.
In women, Lynch syndrome is associated with endometrial and ovarian cancers.
Any patient, especially a non-smoker who presents with TCC of the upper tracts before age 60, should be suspected of having Lynch syndrome. Diagnosis is established by clinical criteria, tumor tissue testing, and genetic evaluation.
Other cancers: colorectal cancer, brain, skin, stomach
A 12-year-old boy is seen with complaints of severe burning pain in his extremities. He is told that he had a congenital disorder and that the disorder would one day affect his kidneys and heart. The examination also reveals the presence of numerous dark skin lesions over the lower trunk. Biopsy of the lesions reveals an excess of ceramide. What is the diagnosis?
Fabry disease is a very rare, x-linked lysosomal storage disorder. There is a deficiency of enzyme alpha-galactosidase, which leads to the accumulation of ceramide in blood vessels, nerves, and the heart.
The disorder eventually affects the heart and kidneys. Angiokeratomas are common painless lesions around the umbilicus and lower abdomen. Fatigue and neuropathy are the most common presenting symptoms.
Other features of the disorder include the inability to gain weight, vertigo, and tinnitus. Diagnosis is made by chromosomal analysis.
Treatment is via enzyme replacement. Expense remains a barrier for most people afflicted with this disorder. Antiplatelet agents are generally used, but sometimes warfarin is needed to prevent strokes.
Mutation Type of SCD Anemia
This patient has likely presented with a vaso-occlusive crisis due to sickle cell disease. In sickle cell disease (SCD), a point mutation causes a single amino acid replacement in beta- chain (substitution of glutamic acid with valine), resulting in the formation of HbS in place of HbA, which causes extravascular and intravascular hemolysis.
The type of point mutation in SCD is called a missense mutation in which a single nucleotide substitution results in changed amino acid. In SCD specifically, an A to T mutation (GTG>GAG) at the sixth codon of the beta-globin gene occurs, leading to the production of a defective form of Hb.
The defective form of Hb in the presence of low O2, high altitude, or acidosis precipitates sickling (deoxygenated HbS polymerized), resulting in anemia due to hemolysis and vaso-occlusive disease.
A missense mutation is called conservative if a new amino acid is similar in chemical structure.
Birt Hogg Dube
Birt-Hogg-Dube syndrome is an autosomal dominant syndrome of fibrofolliculomas, multiple renal tumors, and pulmonary cysts. It is associated with 17p12q11 abnormality involving the folliculin protein.
Associated renal tumors are usually chromophobe carcinomas or oncocytomas. Oncocytomas will have an eosinophilic granular cytoplasm on fine-needle aspiration and will show minimal pleomorphism.
Patients usually have multiple tumors with a mean of 5.3. If a patient has multiple renal tumors, Birt-Hogg-Dube syndrome should be considered.
Tuberous sclerosis is associated with angiolipomas of the kidney. Von Hippel-Lindau disease is associated with renal cell carcinoma. Beckwith-Wiedemann syndrome is associated with somatic overgrowth and frequently has associated renal anomalies including nephromegaly and collecting system abnormalities.
A 17-year-old woman presents with three protrusions of the left lateral ankle that appear only while standing for long periods. Physical examination is notable for three 0.5 cm skin-colored papules protruding from her left lateral heel while she stands, which disappear after she sits. Given the likely diagnosis, which of the following has an association with this patient’s condition?
Piezogenic pedal papules are small papular herniations of subcutaneous tissue on the heel that occur only upon standing or with the application of pressure.
They do not appear to have a hereditary transmission in most patients.
No direct link has been made to a specific connective tissue defect or condition.
However, the literature has noted an association in groups of patients with Ehlers–Danlos syndrome and Prader-Willi syndrome presumably due to weakness in collagen, although no specific mechanism has been proven.
A young patient presents to the clinic for sparse and thin hair. The detailed evaluation confirms the diagnosis of a condition with hair fragility, beaded hair, and patchy dystrophic alopecia. Which of the following examination findings are most likely consistent with the diagnosis?
In severe forms of monilethrix, other hairy areas such as the eyelashes, the eyebrows, or the secondary sexual hairs may be involved.
Perifollicular abnormalities are usually associated with the hair shaft fragility. Most commonly found on the occiput, these perifollicular abnormalities range from perifollicular erythema to large hyperkeratotic follicular papules.
Monilethrix may as well be associated with rare ectodermal symptoms such as syndactyly, cataracts, dental abnormalities, and nail abnormalities (koilonychia).
A 59-year-old man presents to the clinic for a rapidly growing mole on the right shin. Full-thickness excisional biopsy of the lesion is consistent with cutaneous malignant melanoma with a Breslow thickness of 3.2 mm and a negative margin of 0.2 mm. After discussion of the diagnosis, the patient undergoes wide local excision and sentinel lymph node biopsy that reveals a tumor thickness of 2.4 mm and 1 out of 4 sentinel lymph nodes positive for melanoma involvement. PET/CT reveals no evidence of disseminated disease. Which of the following is the next best step in the evaluation of this patient?
BRAF mutational testing is recommended for high-risk stage III cutaneous melanoma for patients in whom BRAF targeted therapy might be an option for adjuvant treatment.
Patients with BRAF mutation can be considered for targeted BRAF inhibitor drugs in the adjuvant setting.
BRAF mutations are identified in over half the patients with cutaneous malignant melanoma.
Serum LDH levels are indicated in patients with disseminated or metastatic disease.
A 10-year-old male presents with skin lesions at the side of the neck, which have a ‘plucked-chicken’ appearance. After detailed evaluation and analysis, it is discovered that he has a mutation of the ABCC6 gene. He also has a recent onset metamorphopsia in one eye. What is most likely to be the cause of this patient’s metamorphopsia?
A. Retinal vasculitis with cystoid macular edema
B. Choroidal neovascular membrane
C. Rhegmatogenous retinal detachment
D. Epiretinal membrane
The features of the patient suggest a diagnosis of pseudoxanthoma elasticum (PXE).
Angioid streaks, which are crack-like breaks in the Bruch membrane, are commonly seen in pseudoxanthoma elasticum (PXE).
Choroidal neovascularization is an essential complication of angioid streaks, which can cause metamorphopsia.
Other ocular features of PXE include peau d’orange appearance of the retina usually at the temporal macula, optic disc drusen, macular pattern dystrophy, and crystalline bodies, some of which may have a tail like a comet.
Familial Melanoma Gene Change
Familial melanoma is associated with a point mutation in the CDKN2A locus at 9p21.
Most melanomas have multiple chromosomal alterations. This feature differentiates melanomas from nevi.
A malignant melanoma developing in healthy skin is said to arise de novo without evidence of a precursor lesion.
A 65-year-old man presents to the office for evaluation of multiple skin lesions on his back. The patient noticed these skin lesions when he was taking a shower and felt irregular skin nodules on his back. He has a past medical history of hypertension, diabetes mellitus, and osteoarthritis. On examination, there is the presence of multiple lesions with a dull, waxy, verrucous surface, giving a ‘‘stuck on’’ appearance. Which of the following is the most likely cause of the patient’s disease?
A. Activating mutation in the fibroblast growth factor receptor-3
B. Loss of function mutation in the fibroblast growth factor receptor-3
C. E6 and E7 oncoproteins
D. Loss of RB1 gene
his patient most likely has seborrheic keratosis. Seborrheic keratosis is a common type of epidermal tumor that is prevalent throughout middle-aged and elderly individuals.
Seborrheic keratosis results from benign clonal expansion of epidermal keratinocytes. There is believed to be a genetic component for the development of a high number of seborrheic keratosis lesions.
Activating mutations in the tyrosine kinase receptor known as fibroblast growth factor receptor-3 (FGFR3) are common in cases of sporadic seborrheic keratosis and are believed to be what drives the growth of this benign tumor.
Under the microscope, seborrheic keratosis typically shows a proliferation of keratinocytes with keratin-filled cysts.
A 10-year-old boy is brought in by his mother due to knee pain and frequent dislocation. The patient was born via an uncomplicated delivery and has met all developmental milestones. On examination, there is decreased muscle bulk in his limbs, swan-neck deformity of the digits, and darkened pigmentation around the iris. An x-ray of the pelvis shows bilateral iliac horns, which confirms the diagnosis. Which of the following would be most likely found on further evaluation of this patient?
A. Dandy-Walker malformation
B. Hepatic cirrhosis
C. Dysplastic fingernails
D. Cardiac arrhythmias
The presence of bilateral iliac horns is pathognomonic for nail-patella syndrome (NPS). Nail changes are the most common clinical manifestation of NPS, which may include absent, hypoplastic, or dystrophic finger and toenails.
The classical clinical tetrad of NPS includes elbow deformities, presence of iliac horns, and absent or hypoplastic patellae.
Both surgical and non-surgical treatment options exist for patients with NPS and knee symptoms.
A 5-year-old boy presents to the office with complaints of nose bleeding. This is the third episode of nose bleeding in the last 6 months. His parents also state he has recurrent skin infections for which he has to take antibiotics. He denies any fever, weight loss, abnormal body movements, or lumps/bumps in the body. On examination, his vitals are stable. Abdominal and chest examinations are normal. His hairs are silver-colored, and his eyes are pale. Complete blood count with differential shows pancytopenia. Bone marrow smear shows giant lysosomes. What is the molecular abnormality associated with this condition?
A. Arylsulfatase A deficiency
B. Mitochondrial DNA mutation
C. Microtubule polymerization defect
D. Defective intestinal absorption of copper
Chediak-Higashi syndrome results from a microtubule polymerization defect causing dysfunction of leukocyte lysosomes and impaired immunity.
It is characterized by oculocutaneous albinism, easy bruising, abnormal functions of the natural killer cells, and recurrent pyogenic infections and is a result of a mutation in the lysosomal trafficking regulator (LYST) gene.
Patients often die before the age of 10 years.
The best treatment is allogenic bone marrow transplantation.
A 3-year-old child is seen in the clinic with complaints of diarrhea, itchy skin, and difficulty walking. The mother reports that she noticed these changes soon after the child started to walk in one year. The child eats a well-balanced diet. The height and weight of the child are below the tenth percentile. Relevant investigations are carried out. In the next appointment, the pediatrician informs the mother that the child has a genetic condition resulting in decreased absorption of neutral amino acids. The mother asks the pediatrician regarding likely complications of this disease. Which of the following is the possible complication of the disease?
A. Liver failure
B. Chronic kidney disease
C. Seizures
D. Pulmonary fibrosis
Hartnup disease is characterized by a defect in gastrointestinal uptake of neutral amino acids.
Neurological complications like seizures, psychosis, and delirium can occur in the patient.
The patients can also have skin hyperpigmentation and dryness at the site of eruptive lesions.
Short stature and below-average school performance have been reported in patients.
Eye exam reveals outpouchings of Descemet membrane in the central cornea
Fuchs endothelial dystrophy (FED)
Progressive worsening vision, corneal edema, outpouchings of Descemet membrane in areas of endothelial cell loss, and thickening of Descemet membrane.
FED can be caused by a dysfunction of the endothelial pump mechanism.
Dysfunction of the endothelial pump impairs corneal deturgescence and causes fluid retention in the cornea.
Channelopathies frequently involved, such as those caused by AD mutations in the SLC4A11 gene.
A newborn infant weighing 2.6 kg is cyanotic with saturation in 80% on room air. A chest x-ray demonstrates a boot-shaped heart, and echocardiogram shows tetralogy of Fallot with severe pulmonary valve stenosis. A patent ductus arteriosus is noted arising off of the underside of the right innominate artery. Which of the following syndromes is most likely to be present in this patient?
A. Williams syndrome (7q11.23 deletion)
B. DiGeorge syndrome (22q11 deletion)
C. Turner syndrome (XO)
D. Noonan syndrome (PTPN11)
The infant described has tetralogy of Fallot with a right aortic arch as defined by a right innominate artery.
The most common genetic syndrome associated with tetralogy of Fallot and a right aortic arch is DiGeorge syndrome.
DiGeorge syndrome can be diagnosed the majority of times by FISH for the 22q11 deletion.
Noonan syndrome is most commonly associated with supravalvular pulmonic stenosis. Turner syndrome is associated with left-sided lesions (coarctation of the aorta, bicuspid aortic valve). Williams syndrome is associated with supravalvular pulmonary and aortic stenosis.
A 25-year-old woman presents to the office for evaluation of a midline neck swelling. The patient states the swelling first appeared 8 months ago and has progressively increased in size. The patient has no complaints of palpitations, weight loss, heat intolerance, or sleeping difficulty. On examination, a 2x2 cm nodular swelling is present over the upper pole of the thyroid gland. Ultrasound of the swelling shows a well-demarcated mixed cystic and solid lesion. After a detailed evaluation, the nodule is resected. Histopathology of the nodule is shown in the exhibit. Presence of which of the following genetic alterations is most likely put the patient at an increased risk of developing malignancy?
A. Rearrangement of the PAX8-PPAR gamma 1
B. Trisomy 21
C. Translocation between chromosome 9 and 22
D. Trisomy 13
This patient has a follicular adenoma. Follicular adenomas are one subset of benign neoplasms that can occur in the thyroid gland or ectopic thyroid tissue. They typically present as a solitary thyroid nodule or in association with nodular hyperplasia or thyroiditis.
Follicular adenomas exhibit rearrangement of the PAX8-PPAR gamma 1. PAX8 helps follicular cell differentiation by encoding a nuclear protein product necessary for the transcription of thyroid-specific factors.
Genetic rearrangement of the PAX8-PPAR gamma 1 gene causes loss of follicular growth inhibition, thus facilitating the development of follicular neoplasms.
PAX8-PPAR gamma 1 has a high predictive value for differentiated thyroid cancer.
A 9-month old boy was referred to dermatology because of severe episodes of sunburns after minimal sun exposure. Examination showed a cachectic boy having a long face, a small protruding chin, and big ears. The boy also had enophthalmia and hypotonia of the muscles. What diagnosis should be first considered?
A. Xeroderma pigmentosum
B. Cockayne syndrome
C. Trichothiodystrophy
D. Rothmund Thomson syndrome
Cockayne syndrome belongs, like xeroderma pigmentosum and trichothiodystrophy, to the group of diseases that affect DNA repair by nucleotide excision (NER).
The most common form of Cockayne syndrome (type I) occurs during the first year of life.
Children have very thin skin and sensitive to ultraviolet rays. The slightest exposure causes severe sunburns that can sometimes leave scars or hyperpigmented spots.
Dysmorphic features include a long face, a small protruding chin, and big ears. Enopthtalmia is common because of the disappearance of the fat that is normally behind the eyeball. The muscles are often insufficiently tonic (hypotonia), and the reflexes are abnormal.
Children with the most common form of Cockayne syndrome (type 1) have very good communication skills for a very long time despite the illness. Their socialization and schooling should be encouraged as much as possible. On the other hand, children with the severe form (type 2) have little possibility of interaction with their surroundings because of the severity of sensory and intellectual impairment. The lives of patients with a lighter form (type 3) are close to normal early on, and children attend formal schooling.
Trichothiodystrophy
(TTD) is a rare inherited, genetic disease characterized a broad spectrum of abnormalities. Patients with different manifestations are linked together by the common feature of short, dry, brittle, sulfur-deficient hair which has a characteristic tiger tail pattern under polarizing microscopy. Typically, patients are born pre-term and with low birth weight. Maternal pregnancy complications are common. Infants may be born with a shiny parchment-like covering on the skin that peels off over several days to weeks (collodion membrane). Through childhood they may have developmental delay or intellectual disability, short stature with poor weight gain, dry, scaly skin (ichthyosis), eye abnormalities (the most common being congenital cataracts), recurrent infections and bone abnormalities. Nearly half (42%) of patients with TTD have extreme sensitivity to ultraviolet radiation (UV) and develop blistering burns on minimal exposure to UV.
Mutation associated with Alagille Syndrome
Alagille syndrome (ALGS) is an autosomal dominant disorder with a wide spectrum of penetrance.
Offspring of an individual with the condition have a 50% chance of inheriting a gene mutation, whereas, among those affected with ALGS, 50% to 70% of individuals have a mutation de novo.
Variants of JAG 1 Notch ligand (chromosome 20p12.2), which encodes protein ligands for the NOTCH2 receptor (chromosome 1p11-p12) account for 94% to 96% of ALGS cases, while variants in NOTCH2 cause around 1% to 2%.
No correlation has been found between a specific mutation and expressed phenotype.
A 16-year-old male presents with delayed puberty. He says that his friends at school have all surpassed him in height and have started to grow facial hair. He is distressed and says, “I look like a child compared to my friends.” He has no medical problems and takes no medications. There is a history of delayed puberty in his maternal uncle. He is an honors student in his school and is currently the leader of the student council. On examination, temperature is 36 C, blood pressure is 125/83 mmHg, pulse is 81 beats/min, and respiratory rate is 14 breaths/min. His height and weight are in the 25th and 20th percentile for his age, respectively. Oral mucosa examination reveals mild cleft palate. There is no restriction in eye movement, and pupils are equal and reactive to light and accommodation bilaterally. There is no facial hair or body hair. The testicles are present bilaterally and are small, non-tender, and have no swellings or masses. A small amount of pubic hair is present above the penis. Blood tests reveal low levels of gonadotropin-releasing hormone (GnRH), follicle-stimulating hormone (FSH), luteinizing hormone (LH), and testosterone. Testicular volume is 3 mL. He has normal levels of thyroid-stimulating hormone, adrenocorticotropin releasing hormone, and antidiuretic hormone. Magnetic resonance of the brain reveals no abnormalities.
This patient is presenting with delayed puberty, as shown by the lack of development of secondary sexual characteristics, low testicular volume, and low blood levels of GnRH, FSH, LH, and testosterone.
Delayed puberty in males is defined by a testicular volume of less than 4 mL by the age of 14 years old.
There are many potential causes of delayed puberty in males. Defective migration of GnRH cells during fetal development is a possible cause of delayed puberty and is known as Kallman syndrome.
Kallman syndrome is an X-linked form of hypogonadotropic hypogonadism. It is defined by defective migration of GnRH cells and defective formation of the olfactory bulb during fetal development. Patients present with delayed onset of puberty with low levels of GnRH, FSH, LH, and testosterone. These patients will also exhibit anosmia (loss of the ability to smell), due to defective olfactory bulb formation. Anosmia is the primary differentiating sign between Kallman syndrome and other causes of delayed puberty.
A 7-year-old girl with cutaneous nodules and cafe-au-lait spots is brought to the hospital with a new constant headache. An MRI reveals a suspicious mass. Which of the following is most likely responsible for the patient’s symptoms?
A. Medulloblastoma
B. Optic nerve glioma
C. Vestibular schwannoma
D. Meningioma
The patient has neurofibromatosis type 1, also called von Recklinghausen disease.
It is characterized by cafe-au-lait spots, subcutaneous tumors, gliomas, bone cysts, Lisch nodules, macrocephaly, epilepsy, meningiomas, precocious puberty, and pheochromocytomas.
It is an autosomal dominant condition caused by a mutation of the NF1 gene.
Optic nerve gliomas are the most common brain tumor in NF1 patients.
Optic nerve sheath meningioma can be differentiated by MRI demonstrating diffuse, tubular thickening of the optic nerve sheath encasing the optic nerve, producing a characteristic “tram track” sign on the axial section or a “doughnut” sign on the coronal section. Choroidal meningiomas are uncommon in NF-1 patients. Arachnoid hyperplasia is a meningeal response associated with gliomas of the anterior optic pathway when these gliomas invade the leptomeninges.
Medulloblastomas are the most common brain tumor in children in general.
Regular newborn screening recognized a child with elevated levels of phenylpyruvate and phenyl lactate in the blood. Despite managing the child with a restricted diet, evidence of developmental delay became apparent. Supplementation with which of the following would be beneficial to the child?
A. Tyrosine
B. 5-Hydroxytryptophan
C. Melanin
D. Alanine
The child has nonclassical phenylketonuria (PKU). Classical PKU is due to a defect in phenylalanine hydroxylase, leading to the accumulation of phenylalanine derivatives. In nonclassical PKU, the required cofactor for the phenylalanine hydroxylase reaction, tetrahydrobiopterin, is deficient. Seratonin requires the cofactor tetrahydrobiopterin for the synthesis. Giving 5-hydroxytryptophan bypasses the block in serotonin biosynthesis, and would have to be a supplement for these children.
These interfere with amino acid transport into the brain and can lead to cognitive disorders if not treated, usually, by a low-phenylalanine diet. Signs include delayed developmental milestones, microcephaly, hypopigmentation, hyperactivity/behavior problems, seizures, and a musty odor to skin and urine. Phenylketonuria is caused by a deficiency of the enzyme phenylalanine hydroxylase and has an autosomal recessive inheritance.
If the heel stick is positive for PKU, further blood and urine samples are obtained to confirm the diagnosis.
Most common cancer in FAP other than colorectal cancer?
Duodenal cancer occurs in 4-12% of FAP patients
A 25-year-old male presents for the evaluation of multiple new soft tissue swellings on his forearms and trunk. He was diagnosed with adenocarcinoma of the colon at 16 years of age and continues to receive appropriate treatment. On examination, the soft tissue swellings are soft, nontender, subcutaneous, and exhibit the ‘slippage sign.’ His mouth is crowded with teeth, and they overlap each other. A bone scan reveals multiple osteomas of the skull and mandible. Which of the following is the most likely diagnosis?
A. Bannayan-Riley-Ruvalcaba syndrome (BRRS)
B. Gardner syndrome
C. Benign symmetric lipomatosis (Madelung disease)
D. Neurofibromatosis 2
Lipomas typically present as soft, solitary, painless, subcutaneous nodules that are mobile and not associated with epidermal change. A characteristic “slippage sign” may be elicited by gently sliding the fingers off the edge of the tumor. They are typically slow-growing and grow to a final stable size of 2 to 3 centimeters. However, they are occasionally greater than 10 centimeters and referred to as “giant lipomas.” Lipomas may appear anywhere on the body but tend to favor the fatty areas of the trunk, neck, forearms, and proximal extremities. Multiple lipomas may be the presenting feature of a variety of syndromes. This patient most likely has Gardner syndrome.
Gardner syndrome is due to autosomal dominant mutations in the adenomatous polyposis coli (APC) gene. Almost all patients develop adenocarcinomas of the gastrointestinal tract. Cutaneous changes include multiple lipomas or fibromas. Other associated findings include congenital hypertrophy of pigment epithelium of retina, osteomas of the skull, maxilla and mandible, supernumerary teeth, and various malignancies, including papillary thyroid carcinomas, adrenal adenomas, and hepatoblastomas.
Gardner syndrome is caused by a mutation in the adenomatous polyposis coli (APC gene), located in chromosome 5q21 (band q21 on chromosome 5). This gene is also mutated in familial adenomatous polyposis (FAP), a more common disease that also predisposes to colon cancer.
Bannayan-Riley-Ruvalcaba syndrome (BRRS) is due to PTEN gene mutations and may represent a pediatric form of Cowden syndrome. Clinical findings include multiple lipomas, intestinal hamartomas, genital lentigines, macrocephaly, and mental retardation. The neurofibromas found in neurofibromatosis 2 would not demonstrate the slippage sign seen in lipomas.
A 3-week-old infant is being evaluated in the neonatal intensive care unit for jaundice. Physical examination reveals a soft systolic murmur with radiation to the axilla. Lab work shows direct hyperbilirubinemia at 0.6 mg/dL, and chest x-ray demonstrates a T11 butterfly vertebra. Which of the following is the most likely underlying etiology for this patient’s presentation?
A. Hepatic vein occlusion
B. Extra hepatic biliary dilatation
C. Polycystic liver disease
D. Paucity of intrahepatic bile ducts
D
The infant has jaundice due to cholestasis. The physical examination reveals a heart murmur that is consistent with peripheral pulmonic stenosis. This is the most frequent cardiac condition found in patients with Alagille syndrome (ALGS).
The finding of a butterfly vertebra on chest x-ray is the third major clinical feature guiding towards the diagnosis of ALGS.
Patients with hepatic vein occlusion and congenital cytomegalovirus can present with signs and symptoms of cholestasis; however, associated cardiac conditions or skeletal abnormalities are not commonly seen with this condition.
The diagnosis of ALGS can be challenging due to the variability of clinical manifestations ranging from no symptoms to life-threatening, even among individuals from the same family that share the same mutation. Most patients present with jaundice or cardiac-related symptoms. Stenosis and congenital paucity of intrahepatic bile ducts represent the underlying etiology for jaundice in this patient.
A preterm male baby is born at 33 weeks of a consanguineous marriage to a multigravida mother by an emergency cesarean section for severe polyhydramnios. There is a history of previous two miscarriages due to unknown reasons. By 7th postnatal day, the baby has lost 12% of his birth weight. There is no history of diarrhea or vomiting. Examination findings are suggestive of symmetrical growth retardation. Chest x-ray and renal ultrasound are normal. ABGs shows metabolic alkalosis, and serum electrolytes reveal hypochloremia, hyponatremia, hypokalemia, and normocalcaemia. Urinary sodium, chloride, and potassium are high, and 24-hour urinary calcium comes out to be normal. What is the most likely diagnosis?
A. Renal tubular acidosis type 2
B. Gitelman syndrome
C. Liddle syndrome
D. Neonatal Bartter syndrome
Neonatal Bartter syndrome presents as polyhydramnios secondary to intrauterine polyuria leading to preterm delivery.
Laboratory findings show hypokalemic, hypochloremic metabolic alkalosis with increased urinary potassium and chloride and increased plasma renin and aldosterone. The blood pressure stays normal.
The pathophysiological basis of the disease can be explained by the mutation involving the Na+/K+/Cl cotransporter (NKCC2). This results in salt and water loss resulting in activation of the renin-angiotensin-aldosterone system secondary to volume depletion.
Patient with Gitelman syndrome will have low 24-hour urinary calcium levels as opposed to Bartter syndrome.
A 34-year-old female presents for a follow-up visit. She visited last month with complaints of oligomenorrhea, headache, abdominal pain, arthralgias, and fatigue. She was given symptomatic treatment and advised to come for a follow-up. The patient still has the same symptoms and has now noticed a milky discharge from her nipples. Her physical examination reveals appropriate sexual maturity. Menarche was at 12. She is not sexually active. Her investigations reveal serum calcium of 11 mg/dl. Her blood pressure is 145/90 mmHg. What is the most likely mechanism involved in this process?
A. Autosomal dominant inheritance involving menin gene
B. Autosomal dominant inheritance involving RET gene
C. Autosomal recessive inheritance involving menin gene
D. Autosomal recessive inheritance involving RET gene
The patient’s hypercalcemia, likely due to elevated parathyroid hormone, and hyperprolactinemia, probably due to prolactinoma, suggest multiple endocrine neoplasia (MEN) Type I. It includes a varying combination of endocrine and non-endocrine tumors.
Multiple endocrine neoplasia (MEN) type I syndrome involves a menin gene mutation. The clinical diagnostic criteria for multiple endocrine neoplasia (MEN)1 syndrome include the presence of two endocrine tumors that are parathyroid, pituitary, or gastrointestinal tract tumors. Multiple endocrine neoplasia (MEN) type I gene mutations present with an autosomal dominant inheritance pattern.
Treatment is multidimensional and focused on the primary manifestations. Subtotal or total parathyroidectomy is indicated for parathyroid adenoma. Surveillance is needed to monitor progress. Genetic counseling is indicated for patients.
Multiple endocrine neoplasia (MEN) type II gene mutations involve the RET gene and are predominantly inherited in an autosomal dominant pattern.
A 3-day-old female, full-term, born without complication to healthy parents, presents with feeding difficulty and abnormal movements. She developed seizure-like activity in the emergency department, and phenobarbital is started. A complete blood count is within normal limits, and blood culture is negative. On physical examination, her vitals are within normal limits. Neurologic examination shows absent suckling reflex. Moro reflex, palmar reflex, plantar reflex, and gag reflex are present. The remainder of her physical examination is unremarkable. Magnetic resonance imaging shows an immature sulcation pattern with the thickening of the cortex. A genetic study reveals a mutation in the LIS1 gene. Which of the following microscopic features is most likely to be present in this patient’s cerebral cortex?
A. Cerebral cortex with six layers
B. Cerebral cortex with four layers
C. Oligodendrocytes with high mitotic activity
D. Cystic space contains necrotic cell debris and macrophages
Lissencephaly is a spectrum of severe brain malformations, including agyria (absent gyri), pachygyria (broad gyri), and subcortical band heterotopia. In lissencephaly, literally means smooth brain, the surface of the brain appears smooth. It is caused by a defect in neuronal migration during embryonic development between 12 and 24 weeks of gestation and results in the absence of normal development of brain gyri and sulci.
LIS1 is responsible for classic lissencephaly (type 1). Examination of the brain in type I lissencephaly shows a cerebral cortex with four layers instead of six layers in normal patients.
A six-layered cortex is associated with DCX gene mutation, compared to lissencephaly caused by LIS1 mutations.
Lissencephaly has different levels of severity and symptoms. Symptoms may include seizures, feeding difficulty, muscle spasm, mental retardation, severe psychomotor impairment, failure to thrive, developmental delays, and sometimes hands, fingers, or toes anomalies. However, some children may develop normally with a mild learning disability.
What is are the NCCN guidelines for offering genetic screening for prostate cancer?
The NCCN guidelines recommend germline testing for all patients with metastatic disease or where there is a father or brother with prostate cancer before age 60, a first degree relative with breast cancer before age 50 and for patients with Ashkenazi Jewish descent.
Germline testing is used for treatment optimization but is most helpful in family counseling to modify screeinings for family members who may be at higher risk.
Specific germline mutations recommended by the NCCN for prostate cancer include ATM, BRCA1, BRCA2, CHEK2, PALB2, PMS2, MLH1, MSH2, and MSH6.
We also suggest adding the following if possible: EpCAM, HOXB13 (especially in African Americans), FANCA, P53, and NBN
A 25 year-old-male presents with complaints of abdominal cramping for the past two weeks. He has also had episodes of vomiting, along with two episodes of bloody stools. On physical examination, dark brown colored spots are seen on the lips, gums, and the inner lining of the mouth, along with both hands and feet. He explains that his father has similar spots on his face and skin. Which investigation is most likely to help in the diagnosis of this patient?
A. Colonoscopy
B. Blood culture
C. Biopsy of mucocutaneous spots
D. Barium swallow
The presence of family history, mucocutaneous pigmentation, and equal or more than two hamartomatous polyps is a classical triad of Peutz–Jeghers syndrome. Colonoscopy can help in the diagnosis of hamartomatous polyps.
Peutz-Jeghers syndrome (PJS) can present with mucocutaneous pigmented macules. These flat dark blue to brown lesions are distributed on the lips, perioral areas, buccal mucosa, eyes, nostrils, fingertips, palms, soles, and perianal areas.
Patients with PJS are at a greater risk for gastrointestinal intussusception with bowel obstruction due to the development of benign hamartomatous polyps in the GI tract.
Hamartomatous polyps in the gastrointestinal tract of patients with PJS may cause intussusception, obstruction, repeated bouts of abdominal pain, bleeding, and rectal prolapse. All individuals who meet more than two out of three diagnostic criteria should be evaluated for the germline mutation in the STK11 gene for confirmatory purposes.
A 16-year-old male presents for a well-child check. His height is at the 80th percentile, weight at the 50th percentile, and mid-parental target height is 50th percentile. He has always had some difficulty with verbal processing and language delay but is in mainstream classes at school. He has a fine-intention tremor. On pubertal exam, he has Tanner IV pubic hair and 3 mL testes bilaterally. Why do patients with these findings need to be followed for many years?
A. Increased risk of schizophrenia and epilepsy
B. Increased risk of germ-cell tumors and breast cancer
C. Increased risk of endocarditis and valvular heart disease
D. Increased risk of peripheral vascular disease and hypertension
Patients with Klinefelter syndrome are at an increased risk for germ cell tumors and breast cancer.
The risk of breast cancer is 20 times that of healthy males.
Other cancers also common in Klinefelter include leukemias, lymphomas, and gonadal tumors. Non-cancerous conditions include osteoporosis and deep vein thrombosis.
Patients often present with an initial complaint of male infertility.
A 30-year-old man presents to the outpatient department with the complaint of difficulty in hearing when listening to his mobile phone conversation over the past one month. On physical examination, he has air conduction better than bone conduction in both ears, but the Weber test is lateralized to the left side. A magnetic resonance imaging (MRI) of the brain is ordered, which shows contrast-enhancing lesions in both cerebellopontine angle regions, extending to the internal auditory meati on either side, the right side lesion being bigger. What is the likely chromosome affected in this patient?
A. 22
B. 17
C. 13
D. 11
The patient has bilateral vestibular schwannomas.
Bilateral vestibular schwannomas are usually associated with neurofibromatosis type 2.
The neurofibromin 2 gene produces merlin (schwannomin), a tumor suppressor.
The neurofibromin 2 gene is located on chromosome 22.
A 2-year-old girl presents to the hospital with seizures. In the past six months, her mother noted that she used to walk on her own before but now needs assistance, she can only talk in 2-word sentences and has difficulty building a tower of 3 cubes. Her head circumference is at 98th percentile for age. The examination of the head shows frontal bossing. There is spasticity of the extremities as well as ataxia. The MRI shows abnormalities in the bilateral cerebral cortex with predominance on the frontal lobes. Evaluation procedures reveal a defect at chromosome 17q21. Which of the following is this disease associated with?
A. Accumulation of very-long-chain fatty acids in the tissues
B. Reduced function of aspartoacylase
C. Defect in glial fibrillary acidic protein
D. Mutations in proteolipid protein 1
Alexander disease is a progressive, lethal leukodystrophy with three forms. They have in common, a defect at chromosome 17q21.
Pathology shows aggregated glial fibrillary acidic proteins and small stress proteins in astrocytes.
They form intracellular inclusions called Rosenthal fibers.
Infants have megalencephaly, seizures, intellectual disability, and premature death. The juvenile and adult forms are characterized by ataxia, bulbar signs, and spasticity.
A 32-year-old woman with myotonic dystrophy type 1 presents to the clinic for prenatal genetic counseling clinic. Pre-implantation testing is positive for a mutation in the DMPK gene. The number of expansion repeats was 40. Which of the following best describes the inheritance of the disorder in her child?
A. Child will likely present with severe symptoms after birth (congenital myotonic dystrophy)
B. Child will likely have mild symptoms in childhood
C. Child will have mild symptoms in adulthood
D. Child will be asymptomatic as it is pre-mutation
Congenital myotonic dystrophy (CMD) is an autosomal dominant neuromuscular disorder with multisystem involvement. It is a subtype of myotonic dystrophy type 1. Features include severe hypotonia and generalized muscle weakness; myotonia is classically absent in infancy.
CMD is caused by the expansion of trinucleotide (CTG) repeat sequence of the myotonic dystrophy protein kinase (DMPK) gene.
The severity of the disease is correlated with allele size (number of repeats). However, mild cases were reported with long expansions. More than 50 full-penetrance alleles are associated with disease manifestations. In CMD, repeats are usually >1000, compared to <37 in normal individuals, and 38-49 in premutation allele patients (asymptomatic). Offspring of premutation patients can inherit longer repeats, increasing the risk of disease and decrease the age of onset in the next generations. This phenomenon is known as anticipation.
Contrary to the classic (adult), the maternal transmission is up to ~90% of cases, and only 9-12% are paternal. The mechanism is not yet well understood; however, maternal and intrauterine environment are considered contributing factors.
A 15-year-old girl is brought to the clinic with primary amenorrhea. Physical examination shows no acne and no axillary or pubic hair but normal breast development. The examination of the inguinal area shows firm slippery masses bilaterally. External genitalia appears to be female. Karyotype type analysis showed 46 XY genotype. USG abdomen reveals the absence of ovaries, fallopian tubes, uterus, and upper vagina. Lab investigations show serum testosterone 4.5 mIU/mL, serum dihydrotestosterone (DHT) 400 pg/mL, LH 16 mIU/mL, FSH 5.5 mIU/mL, and estradiol 150 pg/mL. What is the most likely molecular defect in this disease?
A. Loss of function mutation in the AR gene
B. CYP21A gene mutation
C. SRD5A2 gene mutation
D. SRD5A1 gene mutation
Androgen insensitivity syndrome (AIS), also known as testicular feminization, is an X-linked recessive condition caused by a mutation in the AR gene.
This condition causes failure of masculinization in the external genitalia of male genotypes.
Affected persons have normal testes with the average testosterone production and its conversion to dihydrotestosterone (DHT).
The testes produce normal amounts of müllerian-inhibiting substance (MIS) or anti-müllerian hormone/factor; affected individuals do not have ovaries, fallopian tubes, a uterus, or a proximal vagina.
Overall, female external genitalia with testes and distal vagina; breast development may be present
A 35-year-old man with a history of alcohol use disorder for 10 years is brought to the emergency department with altered mental status and abdominal distension. He recently immigrated to the US from southeastern Iran and has a family history of bleeding disorders. On exam, he bruises easily and has gingival bleeding. Which of the following is the best initial test to confirm the diagnosis in this patient?
A. Mixing study
B. Ammonia release assay
C. Anti-hemophilic factor assay
D. Inhibitor screening test
B
If a patient is from Southeastern Iran, factor XIII deficiency must be considered in your differential for patients presenting with coagulopathies due to its unique prevalence in that region. Factor XIII is a key clotting factor in the coagulation cascade known for stabilizing the formation of a blood clot.
Common symptoms of factor XIII deficiency include bruising, hematomas, muscle bleeds, and delayed post-surgical bleeds. You must consider this rare factor in your differential among other more common coagulopathies. Common clinical signs for coagulation factor deficiencies clinically include mucosal bleeding, bleeding pre, and post-medical procedures, and hemorrhagic diathesis.
During an ammonia release assay, factor XIIIa transglutaminase activity using a synthetic amine peptide substrate reaction generates ammonia. The creation of ammonia is monitored to evaluate the levels of factor XIII.
First-line detection of factor XIII includes a nitrogen release assay and/or a quantitative functional factor XIII assay.
A 5-year-old boy is brought to the clinic for the evaluation of multiple red and 2 brown patches over the face and his right upper limb. Few of the red lesions were present at birth, and few had a pale halo around them. The patches are not associated with warmth or palpable thrill. His parents are worried about the increase in the number of lesions seen over the years. He is developmentally normal and does not have any systemic symptoms. The child’s maternal uncle had similar complaints in his childhood and died of brain injury secondary to intractable seizures. However, there is no such history of similar complaints in the parents. Genetic studies showed the RASA1 gene mutation. Which among the following examination findings is pathognomonic for this condition and prompt the clinician to conduct further imaging studies?
A. Multiple red patches
B. Brown patches
C. Pale halo around the red patches
D. Red patches located over face
Capillary malformation-arteriovenous malformation (CM-AVM) is a rare, combined vascular malformation, characterized by the presence of multiple capillary malformations/port-wine stain and arteriovenous malformation or fistulas. In the majority, it is attributed to dominantly inherited RASA1 gene mutation with high penetrance and intrafamilial clinical variability or sporadic occurrence. Recently EPHB4 gene mutation has also been proposed as a cause of CM-AVM syndrome.
Port-wine stains are slow-flow vascular malformations and have an atypical appearance in CM-AVM syndrome. Multiple port-wine stains are usually seen over the face, trunk, and extremities of the affected individual, at birth, or later in life. They are multifocal, randomly distributed, pink-to-reddish brown colored, of variable size, with geographical margins and can have a pale/white halo around the lesion. It can be mistaken for a café-au-lait macule due to its brownish hue.
Arteriovenous malformation is a fast flow vascular malformation found in one-third of the patients with CM-AVM syndrome. Depending on the site involved, they can present with various symptoms or life-threatening complications. In CM-AVM, AVM can be present in the skin, muscle, bone, spine, and brain. Pain, bleeding, congestive heart failure, and neurological symptoms are some of the important complications. Although symptoms can develop at any age, they commonly present during infancy or early childhood. Early detection and treatment can prevent associated morbidity or mortality.
Parkes Weber syndrome is characterized by multiple arteriovenous fistulas with bone and soft tissue hypertrophy, usually affecting the extremities. RASA1 gene mutation is seen in the patients having port-wine stains on the skin of the affected limb; hence, it is considered as a clinical variant of CM-AVM.
An 8-year-old female presents with difficulty in walking. There is no history of trauma, fever, and weight loss. She experiences a progressive increase in lower limb weakness. She is unable to mobilize and grasp objects. Examination reveals bilateral cavovarus feet and sensory loss on the dorsum of feet. The clinical picture of her hands is shown in the figure. ESR and CRP levels are within the normal range. The nerve conduction study reveals prolonged latencies in the peroneal, ulnar, and median nerves. Which of the following is the most appropriate genetic mutation associated with this disease?
A. t(X;18)
B. PMP-22
C. NF-1 gene
D. t(9;22)
Inherited peripheral neuropathies are a group of disorders that include hereditary motor and sensory neuropathies (HMSN), hereditary motor neuropathies (HMN), and hereditary sensory neuropathies (HSN) or hereditary sensory, and autonomic neuropathies (HSAN). The commonest entity, HMSN is also known as Charcot-Marie-Tooth disease (CMT).
The clinical picture reveals clawing of fingers (intrinsic minus hands). There is a loss of innervation to intrinsic muscles of both hands, depicting median and ulnar nerve palsy. CMT may present with cavovarus feet, clawing of toes, hip dysplasia, progressive scoliosis, and hand intrinsic muscle wasting.
The CMTs are genetically determined disorders with implications of nearly 100 genes. Over 80% to 90% of the genetic abnormalities are due to copy number variation in PMP22 and mutations in GJB1, MPZ, and MFN2 genes. The frequency of abnormalities in other genes individually is rare. Copy number variation in PMP22 is the commonest cause of CMT. PMP22 is a large gene that is located in the middle of the 1.4 Mb regions in chromosome17p.12. This region is susceptible to frequent genomic rearrangements. Duplication or deletion in PMP22 leads to disease via a gene dosage effect.
Nerve conduction studies help in confirming the diagnosis of neuropathy and categorizing patients broadly into demyelinating and axonal subtypes. Patients with CMT have a progressive clinical course and need periodic monitoring. Composite scoring systems are available to assess longitudinal changes in function/ disability, including natural history and outcome. Ankle foot arthrosis (AFO) and insoles are used as conservative management. Mubarak et al and faldini et al devised operative surgical procedures for the correction of cavovarus feet deformity.
A 6-year-old girl presents with a six-week history of bleeding gums. Her mother also reports that her daughter has been complaining of “tiredness” during this time. The patient was born at 37 weeks of gestation by spontaneous vaginal delivery. She has no other medical problems and takes no medication. Vitals are within normal limits. Physical examination reveals that she is at the 5th percentile for height and 40th percentile for weight. Examination of the hands shows slightly bent thumbs, freckling on the face and chest, and areas of hyperpigmentation on the arms. Laboratory findings reveal a hemoglobin of 7.2 g/dL, a platelet count of 30,000/mm^3, and a white blood cell count of 2800 cells/microliter. What is the most likely underlying cause of this patient’s presentation?
A. Congenital infection
B. Immune deficiency
C. Impaired double-stranded DNA repair
D. Red blood cell enzyme deficiency
The most likely diagnosis is Fanconi anemia. Fanconi anemia is an inherited bone marrow failure syndrome that presents with cytopenia and has an increased risk of malignancy (leukemia). Physical abnormalities associated with Fanconi anemia include short stature, microcephaly, and café-au-lait skin lesions. Other physical malformations may also be present, although these are the most common.
Fanconi anemia is a rare hereditary disorder of DNA repair that, in most cases, is autosomal recessive or X-linked. Mutation of FA genes results in impaired double-stranded DNA repair. To date, more than 23 FA complementation genes (FANC) have been recognized, and all of them are involved in the DNA repair pathway.
Among cytopenias, thrombocytopenia is the first to develop and often the only finding on initial evaluation. Neutropenia and eventually anemia develops, indicating bone marrow failure.
Children with Fanconi anemia also have hypogonadism, strabismus, anomalies of the thumbs and radii, and renal abnormalities such as horseshoe kidneys. Intellectual disability and microphthalmia may also be present.
A 6-month-old male is brought to the office by his parents due to recurrent upper respiratory infections. The parents commented that the patient has had this problem since birth, they also add the patient has not been developing as their two previous children. On physical examination, there is hypotonia, macroglossia, and a systolic III/VI murmur located on the left lower sternal border. Two previous affected siblings had a fatal outcome. What enzyme is most likely deficient in this patient?
A. Alpha-1,4-glucosidase
B. Glycogen phosphorylase
C. Hexosaminidase A
D. Myeloperoxidase
Pompe disease is a deficiency in the lysosomal enzyme alpha-1,4-glucosidase.
It is responsible for digesting glycogen-like material accumulating in lysosomes.
The tissues most severely affected are those that have glycogen stores, including cardiac muscle and peripheral skeletal muscle.
Infants present with cardiomegaly, hypotonia, or lethargy.
A 27-year-old male presents after having been unable to conceive with his wife. They have been trying for two years and recently decided to seek medical help. His physical examination was normal, with no gynecomastia and normal male sexual characteristics. His wife also had a normal physical examination. Further evaluation showed that he has microdeletions in the Y chromosome and germ cell aplasia, causing his infertility. Which region on the Y chromosome is most likely to be affected?
A. Yq11 region
B. 22q11 region
C. 18q2 region
D. 17q29 region
This patient has Sertoli-cell-only (SCO) syndrome. It is also called germ cell aplasia and Del Castillo syndrome.
Patients with SCO syndrome present with normal sexual characteristics and infertility. They are usually between 20 and 40 years of age. Their physical examinations are usually normal. A semen analysis will usually show azoospermia.
Microdeletions in the Yq11 region, also known as the azoospermia factor (AZF) region of the Y chromosome, have been found in some patients with SCO syndrome.
Some patients with SCO syndrome with some level of sperm count can still be considered for testicular sperm extraction (TESE). TESE allows the removal of the sperm from the patient’s testes, and the sperm can be used to fertilize an egg via intracytoplasmic sperm injection (ICSI).
A 29-year-old male with a past medical history of recurrent sinopulmonary infections presents to the fertility clinic with his 26-year-old female partner. They are concerned about their inability to conceive after 15 months of unprotected intercourse. The follicular stimulating hormone level (FSH), luteinizing hormone (LH), and testosterone in the male partner are 7.2 mIU/ml (normal range:1.5-12.4 mIU/ml), 6 mIU/ml (1.8-8.6 mIU/ml), and 710 ng/dl (normal range: 270-1070 ng/dl), respectively. Physical examination revealed a testicular length of 5 cm (normal length: above 4.6 cm), and proximal epididymis is enlarged. What is the next preferred step in the management?
A. Obtain a trans-inguinal testicular biopsy and testicular Doppler sonography
B. Request transrectal ultrasonography and testicular Doppler sonography
C. Request gene sequencing and evaluating the 5-thymidine (5T) allele of CCFTR, abdominal ultrasonography
D. Obtain transrectal ultrasonography and repeat semen analysis
Specific characteristics of men with non-obstructive azoospermia (NOA) are small volume testes, a higher level of FSH, along normal semen volume. NOA will typically relate to the impaired sperm production. On the contrary, a normal testicular length of above 4.6 cm, FSH level of lower than 7.6, and/or semen volume of lower than 0.5 or 1.0mL are most likely present in the obstructive azoospermia (OA). The mentioned laboratory tests are highly predicted in the OA, especially when the physical examination is significant for enlarged proximal epididymis or the absence of vasa deferentia.
Ruling out cystic fibrosis (CF) is the most crucial step in diagnosing Young’s syndrome. Cystic fibrosis shares many similar signs and symptoms of Young syndrome. Men with evidence of congenital obstructive azoospermia, including those with congenital bilateral absence of the vas deferens (CBAVD) should be further evaluated for CF.
Mutations in the CFTR gene are present in up to four-fifth of men with CBAVD, and a significantly lower percentage of 20% of men with congenital unilateral absence of the vas deferens (CUAVD). Moreover, up to 21% of men with idiopathic epididymal obstruction have positive genetic testing for CFTR. Abdominal imaging should be requested in men with vasal agenesis irrespective of the CFTR status.
Cystic fibrosis also manifests with recurrent sino-pulmonary infections and infertility. In the rare cases where the man has laboratory and physical examinations suggestive of OA, without epididymal engorgement, a testicular biopsy might be obtained for diagnostic purposes. Scrotal ultrasound should not be included as a primary routine step in the evaluation of infertile men. Request a testicular color Doppler ultrasound might be recommended in uncommon occasions of sub-optimal physical examination in an obese patient or contraction of the dartos muscle even in a warm room while performing the physical exam.
A 32-year-old Japanese female presents to the clinic for worsening of vision in both of her eyes. Initially, it began with the right eye and then progressed to her left eye. She noted that around a week ago, she had a flu-like illness with associated tinnitus. Serologic testing at the time was positive for HLA-DR1 and HLA-DR4. Her visual acuity today is 20/40 in the right eye and 20/50 in the left eye. Pupils are equal and reactive to light. Intraocular pressure is 12 mmHg in both eyes. On fundus exam, there is circumscribed retinal edema, optic disc edema and hyperemia, and whitened areas of retinal elevation. Optical coherence tomography (OCT) shows multiple cystic spaces in the subretinal space with an intact retinal pigment epithelium. In what phase of this disease would these OCT findings most likely be seen?
A. Prodromal phase
B. Acute uveitic phase
C. Chronic/convalescent phase
D. Recurrent phase
The patient has a classic presentation for Vogt-Koyanagi-Harada (VKH) disease, which is a systemic autoimmune condition. There is an increased prevalence of VKH among certain ethnic groups, with a higher rate in Japanese people. People with HLA-DR1 and HLA-DR4 are at higher risks of getting the disease. The OCT shows that the patient has an exudative, or serous retinal detachment. Exudative detachments are typically managed medically and not surgically.
Exudative/serous retinal detachments typically occur in the acute uveitic phase. The uveitis often begins posteriorly. In this stage, the first sign often includes thickening of the posterior choroid with signs of optic disc edema, circumscribed retinal edema, and multiple exudative retinal detachments following. The inflammation causes the retinal blood vessels to leak, and subretinal fluid develops underneath the retina cause a detachment. This can often be best seen with OCT. The inflammation eventually spreads to the anterior chamber, causing panuveitis.
The treatment for VKH is immunosuppression. Intravenous high dose steroids are usually advised for three days following a steroid taper. Treatment may consist of other immunosuppressants. Referral to rheumatology is typically recommended.
In VKH, the prodromal stage presents much like a viral illness and often lasts three to five days. The patient may have fever, headaches, nausea, dizziness, light sensitivity, tinnitus, and orbital pain. The uveitic phase typically presents with blurry vision along with the previously mentioned uveitic sequelae. The chronic/convalescent stage often occurs weeks after the uveitic stage and may last for months. Patients develop poliosis, vitiligo, and choroid depigmentation. The recurrent stage consists of panuveitis with exacerbations of anterior uveitis. Exudative retinal detachments are uncommon in this stage.
A 2-year-old male patient has been brought to the clinic with feeding difficulties since birth. His birth was through a spontaneous vaginal delivery without any complications. There is a history of failure to thrive, short stature, and inappropriate weight gain. His physical examination reveals right-sided bulbous atrophy, abnormal pinnae with hypoplastic left ear cartilage, asymmetrical nostrils, and a high arched palate. Which of the following is the best diagnostic method to diagnose this patient?
A. Fluorescence in situ hybridization (FISH)
B. 7-dehydrocholesterol (7-DHC)
C. Standard karyotyping
D. CHD7 gene mutation testing
CHARGE is the abbreviation of the characteristic features of this disease, such as coloboma, heart defects, atresia choanae (choanal atresia), growth retardation, genital abnormalities, and ear abnormalities.
A 65-year-old male presents with worsening gait disturbance over the past three months. He describes difficulty with some of his more labor-intensive chores, such as shoveling hay, for the past two weeks. His gait disturbances are related to his bilateral hip adductor muscle weakness, and he has hyperreflexia in the upper extremity bilaterally on the physical examination. A family history also reveals that his mother died at 64, approximately four years after being diagnosed with Alzheimer disease. A mutation in which gene is most likely the etiology of his disease?
A. HTT
B. PSEN1
C. ATXN2
D. C9ORF72
Bilateral involvement of the extremities with both upper and lower motor neuron signs and symptoms is characteristic of many forms of amyotrophic lateral sclerosis (ALS).
Some phenotypes of familial ALS have prominent frontotemporal symptoms, which may have been misdiagnosed as Alzheimer’s in his mother, especially given her relatively rapid deterioration.
A hexanucleotide repeat on C9ORF72 is a gene variant that has been found in approximately 40% of familial ALS, making it the most common.
ATXN2 is a gene that, when mutated, causes spinocerebellar ataxia type 2 and may be a susceptibility factor for the development of sporadic ALS.
A 6-year-old girl is brought to the clinic by her mother with concerns of excess body hair. According to the mother, her daughter started to develop hair in the axillary and pubic area 4 months ago, which has progressively increased in quantity and become thicker. She denies a history of menstrual bleeding. She has no past medical problems and does not take any medications or supplements. Family history is remarkable for a younger sister who required genital reconstructive surgery at birth. The physical exam shows an alert, well-looking, friendly girl. Her temperature is 36.8 C, pulse 90/min, blood pressure 100/60 mmHg, and respiratory rate 16/min. Comedogenic acne is seen on the forehead and nose. Growth parameters reveal weight on the 60th and height on the 90th percentile. The height velocity is 7 cm/year. Sexual maturity staging is stage I for breast and stage IV for pubic hair. The systemic exam is unremarkable. Which of the following findings is most consistent with the patient’s condition?
A. Elevated serum estradiol
B. Suppressed serum TSH
C. Elevated serum 17 hydroxyprogesterone
D. Bony deformities on skeletal survey
This girl has signs of secondary sexual development before the age of 8 years along with accelerated height velocity. This clinical picture is suggestive of precocious puberty.
The presence of excess adult-type pubic hair, axillary hair, and acne in this patient suggests the cause of puberty is an excess production of androgens. Other signs and symptoms of adrenal excess in girls include accelerated linear growth and hirsutism.
Non-classical congenital adrenal hyperplasia is an important cause of precocious pseudopuberty in both boys and girls and occurs due to increased androgen production.
Non-classical congenital adrenal hyperplasia is caused by a deficiency in 21-hydroxylase enzyme (CYP21A2) which is responsible for converting 17-hydroxyprogesterone to 11-deoxycortisol. Unlike classical congenital adrenal hyperplasia, enzyme deficiency in non-classical forms is mild and only leads to excess androgen production. The production of glucocorticoid and mineralocorticoid is maintained so salt-wasting does not occur in non-classical forms.
A 17-month-old boy is brought to the clinic by his parents for regression of motor activity. He started walking when he was 12 months old. For the past two months, he has been clumsy with his movements, and his parents report frequent falls. The parents complain of regression of previously attained cognitive milestones. He has a similar history in his elder sibling, who ultimately died at the age of 5. The patient has received all his childhood vaccination. His vital signs are within normal limits. Further blood testing reveals the build-up of cerebroside sulfate. The patients are concerned about his outcome. What is the preferred explanation for the disease condition?
A. Progressive deterioration of the autonomic nervous system only
B. Progressive involvement of the cardiac and vascular system
C. Complete resolution of the symptoms after treatment
D. Progressive deterioration of overall neurocognitive function and neurodevelopment
The patient most likely has metachromatic leukodystrophy due to a deficiency of arylsulfatase A.
It is an inherited disorder that predominantly affects the central nervous system (CNS) white matter tracts, and it’s cellular components.
This leads to the accumulation of sulfatides, which result in the dysfunction and destruction of the CNS and peripheral nervous system (PNS) myelin sheaths and overall neurocognitive function and neurodevelopment deterioration.
Symptomatic supportive care is needed to address the neurocognitive and neurodevelopment defects as no curative treatments are available.
A 17-year-old male is being evaluated for gynecomastia. Medical and surgical histories are normal. However, the mother noticed him becoming more wobbly during walking. On physical examination, he is alert and awake. Poor coordination is noted during his neurological examination. The pubertal examination is concerning for gynecomastia, small testes, and an undersized penis. Which of the following tests is most likely to reveal the diagnosis in this patient?
A. Brian magnetic resonance imaging (MRI)
B. Karyotype
C. 17-hydroxyprogesterone level
D. Prolactin level
Poor coordination, gynecomastia, small testes, and penis are signs suggestive of Klinefelter syndrome.
Klinefelter syndrome is diagnosed by chromosomal analysis (karyotype).
The majority of patients with Klinefelter syndrome are diagnosed around the age of puberty.
Low androgens with high FSH and LH levels are suggestive of Klinefelter syndrome.
A 51-year-old postmenopausal woman presents for follow-up of a finding of persistent microcytosis. The patient is asymptomatic and denies any bleeding, blood transfusion, surgery, or previous diagnoses of anemia. Her complete blood count shows a hemoglobin of 12.3 g/dL, hematocrit 39.6%, mean corpuscular volume (MCV) 72.1 fl, mean corpuscular hemoglobin (MCH) 22.4 pg and a red cell distribution width (RDW) of 14%. Her iron studies are essentially normal (transferrin saturation 29%, ferritin 84 ng/mL). Hemoglobin electrophoresis shows a pattern of HbA1 97.1%, HbA2 2.6%, HbF 0.3%. The blood smear review describes moderate hypochromia and poikilocytosis with regular codocytes. What is the most likely diagnosis?
A. Thalassemia intermedia
B. Congenital microcytosis
C. Alpha-thalassemia silent carrier
D. Thalassemia minor
The silent carrier state for alpha thalassemia causes only microcytosis and hypochromia without anemia. Codocytes (target cells) are a common finding in all the thalassemic states. The condition is clinically silent and does not require treatment.
Humans have two alpha-globin genes located on each chromosome 16, resulting in 4 a-gene loci (aa/aa). Adult hemoglobin is a mixture of HbA1, HbA2, and HbF. All of these hemoglobins are composed of 2 alpha-globin chains and 2 non-alpha-globin chains (delta in HbA2 and gamma in HbF). The alpha thalassemia carrier state is the result of deletion or inactivation of only one alpha-globin gene (-a/aa).
Hemoglobin electrophoresis is normal in adults with the alpha thalassemia carrier state. Confirmation of the diagnosis requires molecular analysis.
The condition is also referred to as alpha thalassemia minima.
A 24-year-old woman gives birth to a liveborn baby boy with obvious deformities, including fully formed hands attached to his trunk at the shoulder and craniofacial defects. She denies a history of thalidomide use. Given the likely diagnosis and most commonly associated syndrome, what gene mutation is expected in this newborn?
A. FGFR-3
B. ESCO2
C. COL1A1
D. Fibrillin-1
Phocomelia is defined as a truncation of the intercalating segments of the upper or lower extremity, with hands or feet attached directly to the trunk.
About 10% of patients with phocomelia have an underlying clinical syndrome, most commonly Robert syndrome.
Robert syndrome is caused by an autosomal recessive mutation of the ESCO2 (establishment of cohesion 1 homolog 2) gene, resulting in syndromic facies and limb truncation defects.
Phocomelia most commonly affects one limb, typically the upper left limb.
A 2-week-old female is brought in with generalized edema and increasing abdominal distension. She was born at 40 weeks of gestation with a birth weight of 2496 g (5.5 lbs). Physical examination shows microcephaly, microphthalmos, epicanthic folds, a narrow slopping forehead, a highly arched palate, large low-set floppy ears, a small midface, and a beaked nose, thin lips, clenched hands, and arachnodactyly. An initial set of lab investigations is shown below.
Patient value Reference range
Serum albumin 0.9 g/dL 3.5-5 g/dL
Serum protein 2.45 g/dL 6.5-8.2 g/dL
Urine protein 4000 mg/dL <100 mg/dL
Renal ultrasound shows enlarged bilateral kidneys. What is the most likely diagnosis?
A. Frasier syndrome
B. Denys-Drash syndrome
C. Pierson syndrome
D. Galloway-Mowat syndrome
Galloway-Mowat syndrome (GMS) is an autosomal recessive disorder. It is characterized by microcephaly with various central nervous system anomalies and early-onset nephrotic syndrome.
Physical findings include microcephaly, microphthalmos, epicanthic folds, a narrow slopping forehead, a highly arched palate, large low-set floppy ears, a small midface, and a beaked nose, thin lips, clenched hands, and arachnodactyly.
The gene for Galloway-Mowat syndrome is not known.
Denys-Drash syndrome (DDS) is characterized by male pseudo-hermaphroditism, Wilms tumor, and early-onset nephrotic syndrome that progresses rapidly to end-stage renal disease. Frasier syndrome (FS) is characterized by progressive glomerulopathy, male pseudo-hermaphroditism, and gonadoblastomas. Pierson syndrome is characterized by congenital nephrotic syndrome/diffuse mesangial sclerosis, microcoria (small pupils), impairment of vision, and muscular hypotonia.
A 3-year-old male presents to the office for bilateral hearing loss. The ears are clean on the physical exam without any signs of infection or obstruction. He was also noted to have a soft midline neck mass on palpation. A CT scan of his head was done, which showed normal middle ear structures but enlarged vestibular aqueducts. What is the inheritance pattern of this syndrome?
A. Autosomal dominant
B. Autosomal recessive
C. X-linked recessive
D. Y-linked
Autosomal dominant causes of hearing loss can be summarized in the acronym WANT CBS (Waardenburg, Apert, neurofibromatosis, Treacher-Collins, Crouzon, brachio-oto-renal, and Stickler). This patient has Pendred syndrome, which is autosomal recessive.
The patient presents with the constellation of bilateral hearing loss, euthyroid goiter, and bilateral vestibular aqueducts consistent with Pendred Syndrome. Pendred Syndrome is inherited in an autosomal recessive manner.
X-linked recessive causes of hearing loss include Alport syndrome and Mohr-Tranebiaerg syndrome. This patient has Pendred syndrome, which is autosomal recessive.
There are no Y-linked causes of hearing loss.
A 17-year-old male patient presents for the evaluation of dark-colored urine. Past medical history is insignificant. Vital signs are pulse rate 78/min, blood pressure 117/68 mmHg, respiratory rate 17 breaths/min, and temperature 98.6 F (37 C). Physical examination reveals scleral icterus. An abdominal examination is unremarkable. Lab shows total bilirubin 3.5 mg/dL, ALT 38 IU/L, AST 65 IU/L, and alkaline phosphate 170 IU/L. Abdominal ultrasound reveals normal liver texture. Urine coproporphyrin levels are elevated. What gene is responsible for the underlying condition?
A. ABCC2
B. CFTR
C. SLCO1B1
D. NPC-1
Rotor syndrome is an autosomal recessive disease and a rare cause of mixed direct (conjugated) and indirect (unconjugated) hyperbilirubinemia. The disease is characterized by non-hemolytic jaundice due to the chronic elevation of predominantly conjugated bilirubin.
Rotor syndrome is an autosomal recessive disorder caused by homozygous mutations in both the SLCO1B1 and SLCO1B3 genes on chromosome 12. These genes provide instructions for making organic anion-transporting polypeptide 1B1 and 1B3 (OATP1B1 and OATP1B3, respectively).
These proteins are found in liver cells and mediate sodium-independent cellular uptake of compounds, including bilirubin glucuronide, bile acids, and steroid and thyroid hormones, as well as numerous drugs, toxins, and their conjugates.
In rotor syndrome, the OATP1B1 and OATP1B3 proteins are abnormally short; therefore, the bilirubin is less efficiently taken up by the liver and removed from the body, causing a buildup of bilirubin in the blood and urine, which results in jaundice and dark urine.
A 54-year-old woman presents to the clinic for a follow up of an abnormal fundoscopic exam at a health fair. The exam is documented, showing the presence of multiple, bilateral ovoid cream-colored chorioretinal lesions scattered radially from the optic nerve with a predominance in the nasal retina. The patient denies a significant change to her vision, though she has been struggling with her vision at night recently, and she feels that some colors are not as intense as usual. Examination reveals normal visual acuity and normal intraocular pressure. There is mild vitritis present, and the fundoscopic exam confirms the previously documented chorioretinal lesions. Which of the following studies is most likely to confirm the diagnosis in this patient?
A. Angiotensin-converting enzyme level
B. HLA-A29
C. Interferon-gamma release assay
D. Rapid plasma reagin
Birdshot chorioretinopathy is a white dot syndrome with bilateral eye involvement that typically affects more women than men.
Birdshot chorioretinopathy is highly correlated with the presence of the HLA-A29 haplotype. Correlation is so high that the absence of HLA-A29 should raise suspicion for an alternative diagnosis.
The classic chorioretinal lesions of birdshot chorioretinopathy are multiple ovoid cream-colored lesions scattered in a birdshot pattern, often radially from the optic nerve with a nasal predominance.
Patients with birdshot chorioretinopathy often present with visual symptoms out of proportion to measured visual acuity. Initial symptoms typically include photopsia, poor night vision (nyctalopia), alterations in color perception, and floaters.
A baby is born by C-section due to cephalopelvic disproportion. Examination of the infant shows the anterior fontanelle is closed with a palpable coronal suture ridging, hypertelorism, and syndactyly of the hand and feet. What syndrome is most likely represented?
A. Crouzon
B. Apert
C. Pfeiffer
D. Muenke
Apert syndrome presents with premature fusion of the coronal suture. These patients have with midface hypoplasia, hypertelorism, syndactyly of hands and feet, hearing loss, and cervical vertebral fusion.
Pfeiffer syndrome features brachycephaly (premature fusion of bicoronal sutures), hypertelorism, maxillary hypoplasia, broad thumbs, great toe, syndactyly, brachydactyly, and hearing loss.
Muenke affects the coronal suture unilaterally or bilateral and features midface hypoplasia, hypertelorism, macrocephaly, and hearing loss.
The only option stated that features syndactyly of the hands and feet with unilateral coronal suture fusion is Apert.
A 6-year-old female is brought to the clinic for nasal congestion, rhinorrhea, sneezing, and a dry cough. Laboratory investigations show marked eosinophilia and increased serum IgE. Her family history is significant for asthma in the mother and eczema in an elder brother. Which of the following chromosomal locations are related to the underlying condition in this case?
A. 3p
B. 1p
C. 11p
D. 5q
Chromosome 5q is linked to the development of atopy, but 3p, 1p, and 11p are not.
Chromosome 5q encodes for IL-4, IL-5, and IL-13, CD14, and beta2 adrenergic receptors.
IL-4 and IL-13 promote IgE switching, and IL-5 promotes the growth of eosinophils.
CD14 is an LPS receptor component, via interaction with TLR4 and may influence the balance between Th1 and Th2 responses to antigens. Beta2-adrenergic receptor regulates bronchial smooth muscle contraction.
Which chromosome controls allergy symptoms?
Chromosome 5q is linked to the development of atopy, but 3p, 1p, and 11p are not.
Chromosome 5q encodes for IL-4, IL-5, and IL-13, CD14, and beta2 adrenergic receptors.
IL-4 and IL-13 promote IgE switching, and IL-5 promotes the growth of eosinophils.
CD14 is an LPS receptor component, via interaction with TLR4 and may influence the balance between Th1 and Th2 responses to antigens. Beta2-adrenergic receptor regulates bronchial smooth muscle contraction.
After surgery of an intraventricular tumor, the pathologist reports a spindle-shaped tumor cell, with narrow rod-shaped nuclei, and abundant collagenous or reticulum background. On immunostaining, it showed positive for epithelial membrane antigen, positive for Vimentin, and positive for S100 protein. What is the most likely genetic mutation of the mass mentioned above?
A. 1p19q mutation
B. IDH-1/2 wild type
C. Chromosome 17 mutation
D. Chromosome 22 mutation
The most common chromosomal mutation of all meningiomas is chromosome 22, specifically with merlin tumor suppressor gene, SIS oncogene, and INI1.
The most prevalent subtype of intraventricular meningioma is a fibrous meningioma.
On immunostaining, fibrous meningiomas typically present with epithelial membrane antigen/vimentin/S100 positive. S100 protein positivity is more common in fibrous meningiomas, compared to other subtypes.
1p19q is seen in oligodendrogliomas, IDH wildtype is seen in primary glioblastomas, and chromosome 17 is associated with the development of neurofibromas.
A 3-year-old boy is seen in the outpatient department with complaints of swollen, red, and tender gums. Past medical history is significant for recurrent skin and upper respiratory tract infections. He has white hair and pale eyes. Complete blood count reveals the presence of pancytopenia. Bone marrow aspiration shows giant lysosomes. Molecular testing shows a mutation in the lysosomal trafficking regulator (LYST) gene. Which of the following symptoms are most likely to be seen in a patient with this disorder?
A. Arrhythmias
B. Clotting dysfunction
C. Eczema
D. Uncontrollable pruritus
B
Chediak–Higashi syndrome is a rare autosomal recessive disorder.
It arises from a mutation of a lysosomal trafficking regulator protein. This leads to decreased phagocytic activity and decreased immunity. Clinical manifestation includes recurrent pyogenic infections, albinism, and peripheral neuropathy.
Clotting dysfunction is a feature of Chediak-Higashi syndrome.
In addition to bruising, dermatological presentations include blonde hair and patches of non-pigmented skin.
A 17-year-old male presents for a routine visit. His BMI and weight are above the 97th percentile, and his height is in the 85th percentile. He has a heart rate of 88 beats per minute, and his blood pressure is 120/81 mmHg. He has no temperature intolerance, no constipation, or diarrhea and denies dry skin. Thyroid function test show TSH 2.3 mIU/L (0.3-4.2 mIU/l), total T4 3.5 mcg/dL (5.5-12.8 mcg/dL) and free T4 1.1 ng/dL (0.76-1.46 ng/dL). Which of the following best identifies the gene implicated in this disorder?
A. TTR
B. MAP3K1
C. SERPINA 6
D. SERPINA 7
This patient has thyroxine-binding globulin (TBG) deficiency based on a normal TSH, free T4, and low total T4, and the protein that is responsible for this is thyroxine binding globulin on TBG. TBG is encoded by a gene called SERPINA 7.
TBG is encoded by SERPINA 7, is a serine protease inhibitor, and is found on the X-chromosome.
Mutations in the SERPINA 7 gene can cause alterations in the level of TBG, which in turn can cause a decrease in total T4 but not in the free T4 as TBG is bound to T4. As such, patients do not experience symptoms since there is no change in the level of free T4.
TTR is responsible for making transthyretin, another major transport protein for thyroid hormones, but this does not result in TBG deficiency. The SERPINA 6 gene is responsible for making corticosteroid-binding globulin, and MAP3K1 is a gene that plays multiple roles in signaling pathways. These two genes are not implicated in TBG deficiency.
A 13-year-old male is brought in for an inability to smell various odors. He denies recent illness or fever. His mother states that he has always had difficulty with smell, which has led him to bathe irregularly due to the inability to discern body odor. A physical exam reveals that the patient has difficulty hearing when tested bilaterally with the finger rub test. Laboratory testing of the patient’s blood reveals elevated levels of phytanic acid. Which of the following genes is most likely defective in the patient?
A. PHYH gene
B. ABCD1 gene
C. PEX1 gene
D. RET gene
The patient presents with a long history of anosmia and progressive deafness, both of which are symptoms of Refsum disease.
The laboratory testing of the patient’s serum shows elevated levels of phytanic acid, a breakdown product of chlorophyll in the diet of animals and humans.
The PHYH gene encodes the enzyme phytanyl CoA hydroxylase (alpha-hydroxylase), which is required for the alpha oxidation of phytanic acids. Deficiency of this enzyme in peroxisomes leads to Refsum disease.
ABCD1 gene mutation is present in patients with X-linked Adrenoleukodystrophy. PEX1 gene mutation is responsible for Zellweger syndrome. Mutations in RET are associated with multiple endocrine neoplasia type II.
During a murder investigation, the coroner identifies that a middle-aged individual suffered from inappropriate, extreme, or erratic behavior and moods, hallucinations, oscillating hypertonia, ataxia, and rapid neurological decline before death, certified as cerebral edema by the neuropathologist. A molecular biology investigation using next-generation sequencing determines a mutation in the BCKDHA gene. The failure to break down isoleucine and other amino acids is associated with the detected genetic mutation. The coroner also receives some mass spectrometry results, which indicate that methylene-cyclopropyl acetyl-CoA was present in a glass of orange juice at the crime scene. In addition to pyruvate, which compound is critical for synthesizing isoleucine?
A. Alpha-ketobutyrate
B. Tryptophan
C. Tyrosine
D. Cysteine
Isoleucine is an essential nutrient because it is not synthesized in the body, but it is synthesized via several steps in plants and microorganisms, starting from pyruvate and alpha-ketobutyrate.
Enzymes involved in this process include acetolactate synthase, acetohydroxy acid isomeroreductase, dihydroxy acid dehydratase, and valine aminotransferase.
Isoleucine is a glucogenic and ketogenic amino acid, and methylene-cyclopropyl acetyl-CoA is derived from the hypoglycin plant toxin and inhibits the ß-oxidation and the catabolism of branched-chain amino acids.
Maple syrup urine disease (MSUD) with late-onset includes neurologic symptoms such as inappropriate, extreme, or erratic behavior and moods, hallucinations, oscillating hypertonia and hypotonia, ataxia, seizures, opisthotonos, and coma.
A 26-year-old man is brought to the clinic with a sad mood and frequent crying. He is known to have schizophrenia on treatment and lives alone. He has no family he knows of. He also says that the voices in his head have stopped, but he cannot help feeling useless and worthless. Examination reveals winking of eyelids, corrugated brows, and elevated screwed nose on the left side. No other neurological symptom is observed. He is prescribed antidepressants, and his medications are altered. Five weeks later, he comes back with no improvement but rather a worsening in his mental state. An MRI reveals a ‘boxcar’ appearance. What is the most likely underlying pathology in this patient?
A. Excess dopamine suppression
B. Atrophy of putamen
C. Cerebellar atrophy
D. Atrophy of subthalamic nuclei
This patient with severe depression and signs of facial muscle twitching that does not resolve on antidepressants and altering antipsychotics, respectively, with no account of family history, and an MRI of a boxcar appearance is most likely a case of Huntington chorea. It is caused by the atrophy of the caudate and putamen.
The classic clinical triad in Huntington disease is (1) progressive movement disorder, most commonly chorea; (2) progressive cognitive disturbance culminating in dementia; and (3) various behavioral disturbances that often precede diagnosis and can vary depending on the state of disease.
The enlargement of the frontal horns often gives a “box” like configuration. This can be quantified by a number of measurements 1. frontal horn width to intercaudate distance ratio (FH/CC) 2. intercaudate distance to inner table width ratio (CC/IT).
Excess dopamine suppression due to antipsychotics can cause acute dystonia. However, stopping the antipsychotics and treating with benztropine should treat this. Cerebellar atrophy is seen in spinocerebellar ataxia. This presents with ataxia primarily along with other symptoms like slowing of ocular saccades, paralysis, etc. Atrophy of the subthalamic nucleus causes hemiballismus, which causes wild flailing of limbs, rather than facial chorea in Huntington chorea.
A 31-year-old man presents to the emergency department with fevers and general irritability. He was found to have a positive urine culture and was started on gentamicin. He presented to his PCP’s office a week later with worsening hearing loss. What is the likelihood that his children will also present with the same genetic hearing disorder?
A. 0% of his children
B. 25% of his children
C. 50% of his children
D. 100% of his children
The patient has a sensitivity to aminoglycosides given the onset of hearing after starting gentamicin. Aminoglycoside-induced deafness is inherited in a mitochondrial manner, meaning only the mother can pass on those genes. Since he is a male, he would pass on the gene to 0% of his offspring.
Hearing loss can present in 25% of children of parents who both have an autosomal recessive gene for hearing loss. Aminoglycoside sensitivity is inherited in a mitochondrial manner.
Hearing loss can present in 50% of children of parents if one of them carries an autosomal dominant gene for hearing loss. Aminoglycoside sensitivity is inherited in a mitochondrial manner.
Aminoglycoside sensitivity is inherited in a mitochondrial manner. If the patient had been female, the rate would be 100%.
Which renal disorder can patients have in conjunction with tuberous sclerosis?
A. Renal adenocarcinoma
B. Renal artery stenosis
C. Angiomyolipoma
D. Nephrotic syndrome
Renal manifestations of tuberous sclerosis include angiomyolipomas and renal cysts.
Angiomyolipomas are seen in 80% of patients and are composed of smooth muscle, adipose tissue, and connective tissue.
Renal failure does occur from bleeding angiomyolipomas and is a common cause of death in these patients.
Rarely, large lesions undergo differentiation to renal cell cancers.
A one-week-old baby is brought by her mother for a check-up. She was born vaginally, and the delivery was uncomplicated. Apgar scores were 8 and 10 at 1 and 5 minutes, respectively. The patient was born with a mass protruding from her left sternocleidomastoid. On examination, the mass is soft, measures 5 cm in diameter, appears to contain fluid, and is unilocular. What underlying condition is most likely present in this newborn?
A. Fragile X syndrome
B. Turner syndrome
C. Von Hippel-Lindau syndrome
D. Angelman syndrome
The mass present in the newborn is most likely a lymphangioma.
Congenital lymphangiomas form due to blockage of the lymphatic system during fetal development, though the cause remains unknown. Cystic lymphangiomas are associated with genetic disorders, including trisomies 13, 18, and 21, Noonan syndrome, Turner syndrome, and Down syndrome.
Lymphangiomas are soft, with varying sizes and shapes, and will typically grow if not surgically excised. When posterior neck lesions are present, there may be an association with Turner syndrome, hydrops fetalis, or other congenital abnormalities.
Lymphangiomas result from congenital or acquired abnormalities of the lymphatic system. The congenital form typically occurs before the age of 5 years. Acquired lymphangiomas occur as a sequela of any interruption of previously normal lymphatic drainage such as surgery, trauma, malignancy, and radiation therapy.
A 13-year-old boy presents because his family has noticed skin changes. He has very light skin and hair compared to his parents and siblings. He also reports that he has become more short-sighted over the last few years, and on ocular examination, astigmatism and reduced visual acuity is found. Given the likely diagnosis, which of the following conditions results from dysfunction, rather than absence, of the cell type involved?
A. Crouzon syndrome
B. Piebaldism
C. Vitiligo
D. Oculocutanous albinism
Oculocutanous albinism is a recessive disorder that affects melanin production without affecting melanocyte numbers or distribution in the skin.
The relative lack of pigment leads to paler skin, hair, and eyes.
The ocular findings of this condition can be debilitating and are divided into refractive and nonrefractive errors.
Vitiligo and piebaldism are hypopigmentation skin conditions caused by autoimmune destruction and patchy absence of melanocytes, respectively Crouzon syndrome is an autosomal dominant branchial arch syndrome affecting the pharyngeal arch that develops into the maxilla and mandible, and is not a hypopigmentation syndrome.
A 3 year old boy has bouts of uncontrollable laughter. He has a hx of imperforate anus s/p correction. Exam shows ID and polydactyly of right foot and left hand. Testing shows GLI3 mutation on chromosome 7. What is the disorder?
A. Holt-Oram syndrome
B. Currarino syndrome
C. Dandy-Walker syndrome
D. Pallister-Hall syndrome
Up to 5% of cases of hypothalamic hamartomas are associated with Pallister-Hall syndrome.
Hypothalamic hamartomas are non-neoplastic tumors of the hypothalamic region consisting of normal neural and glial cells.
Gelastic seizures are uncontrolled bouts of laughter that occur with hypothalamic lesions.
Pallister-Hall syndrome is also associated with other abnormalities such as hypopituitarism, growth hormone deficiency, bifid epiglottis, and genital hypoplasia.
Pallister-Hall syndrome is typically diagnosed in early childhood presenting with seizures, central precocious puberty due to hypothalamic hamartomas, and polydactyly. Not all have seizures.
Pallister-Hall syndrome has an autosomal dominant inheritance pattern. It is caused by mutations in the GLI3 gene in chromosome 7. It is a dysmorphology syndrome involving the hands, feet (polydactyly and syndactyly), larynx, anus, and hypothalamus.
8 year old with multiple cardiac sx for aortic root ectasia and MVP. He has hyperextensible skin and elbows. Appears marfanoid but arm span to height ratio is 1.01. Which gene is most likely to be the culprit?
A. COL1A2
B. COL5A1
C. Fibrillin-1
D. FBN2
Cardiac-valvular Ehlers-Danlos syndrome (EDS) involves an autosomal recessive inheritance pattern and is associated with mutations in the COL1A2 and NMD genes, which code for type 1 collagen. Major clinical criteria include skin hyperextensibility, atrophic scarring, easy bruisability, restricted or generalized joint hypermobility, and progressive cardiac-valvular problems.
Classical EDS involves an autosomal dominant inheritance pattern, and associated mutated genes include COL5A1 and COL1A1, which code for type 5 and type 1 collagen, respectively.
Genetic testing of Marfan syndrome can reveal a mutation in the fibrillin 1 protein, which does not occur in EDS. Elbow involvement is generally spared, and marfanoid habitus is usually more evident in Marfan syndrome, with an arm span-to-height ratio greater than 1.05.
Genetic analysis of a patient with cutis laxa will often identify mutations in the fibulin-5 gene. The skin of patients with cutis laxa is characteristically known to slowly return to its original form from distention, unlike EDS.
A 17-year-old male comes in with the complaint of decreased vision. He also noticed graying and loss of hair recently. There is a family history of similar symptoms in a paternal uncle. Vital signs show pulse rate 82/min, blood pressure 125/70 mmHg, respiratory rate 17/min, and temperature 98.6 F (37 C). Physical examination reveals short stature and loss of muscle mass, most notably in the limbs with sparing of the trunk. He is referred to an ophthalmologist for eye examination who diagnosed a cataract in the right eye. Which of the following malignancies is most commonly found with this condition?
A. Papillary thyroid cancer
B. Renal cell carcinoma
C. Small cell lung carcinoma
D. Sarcomas
Werner syndrome patients appear unaffected at birth and develop normally until the adolescent period or second decade of life, when they start to exhibit signs and symptoms of accelerated aging. Patients develop skin ulcers, cataracts, graying, or even loss of their hair, hypogonadism, and can develop a malignancy.
Up to 50% of the malignancies reported are soft tissue sarcomas, such as schwannoma, rhabdomyosarcoma, undifferentiated pleomorphic sarcoma, leiomyosarcoma, and osteosarcoma of the upper extremities. Patients have also been reported to develop meningiomas, malignant melanoma, and thyroid carcinomas.
Werner syndrome patients are at an increased risk of tumor formation, with up to 10% of patients developing a malignancy. However, the types of tumors that develop are unusual compared to the aging population.
If a Werner syndrome patient develops a malignancy, they should be referred to the appropriate specialist for the staging of the tumor. The life expectancy of a patient with Werner syndrome is around 50 years. Patients usually die from malignancy or cardiovascular complications.
A 17-year-old boy presents with behavioral changes. His family noticed that for the past three months, he has become progressively withdrawn and prefers to stay in his room by himself. He has also become impulsive and aggressive when not given what he wanted. His school performance has dramatically deteriorated. He was a high-performing student previously and now barely passes his subjects. His mother also reports poor appetite, constant fatigue, intermittent vomiting, and increased pigmentation on the neck and axillary areas. On examination, he is not oriented to time but oriented to place and person. Mini-mental status exam score is 20/30. There are no cranial nerve, motor, or sensory deficits. He has a wide-based gait and cannot walk in tandem. A contrast-enhanced MRI reports white matter changes on bilateral parietooccipital regions. Which of the following inheritance pattern does this illness follow?
A. Autosomal dominant
B. Autosomal recessive
C. X-linked
D. Mitochondrial
Adrenoleukodystrophy (ALD) is an X-linked disease with a mutation in the ABCD1 gene.
This gene codes for ALD, a peroxisomal membrane transporter protein. It is associated with high serum very-long-chain fatty acids.
Several different phenotypes and ages of onset of adrenoleukodystrophy exist, and presentation is variable even within the same family.
A progressive adrenal gland dysfunction and loss of myelin are characteristic of this illness. Symptoms include symptoms of adrenal insufficiency, behavioral changes, poor memory, gait disturbances, and progressive dementia.
A 4-month-old male is being evaluated for developmental delay. The infant was delivered by a caesarian section at term due to fetal distress and breech presentation. The patient is the first child for a young couple who are also first degree relatives. The mother did not receive any antenatal care. On physical examination, facial dysmorphism, hypotonia, psychomotor retardation, and hepatomegaly are present. Biochemical analysis show elevated levels of blood pipecolic acid and very-long-chain fatty acids VLCFAs. Impairment of which of the following cellular structure is responsible for the child’s condition?
A. Peroxisome
B. Mitochondria
C. Smooth endoplasmic reticulum
D. Golgi apparatus
Zellweger syndrome, also known as cerebrohepatorenal syndrome, is a very rare inherited disorder characterized by the absence or reduction of functional peroxisomes in cells. It is an autosomal recessive disorder and is one of the disorders included under disorders of peroxisome biogenesis.
Zellweger syndrome is caused by mutations in various genes required for peroxisome biogenesis. The peroxisome is a single membrane-bounded organelle that is involved in degradation (beta-oxidation) of very-long-chain fatty acids (VLCFAs), and catabolism (alpha oxidation) of branched-chain fatty acids, catabolism of amino acids and ethanol, biosynthesis of bile acids, steroid hormones and plasminogen formation which are important in the cell membrane and myelin. It is also involved in the degradation of cytotoxic Hydrogen peroxide.
Zellweger syndrome is thus characterized by abnormal accumulation of very-long-chain fatty acids (VLCFA) as the normal peroxisomal function is markedly reduced or absent. There is an increased accumulation of VLCFA with 26 carbons, an increased ratio of C26 to C22 very-long-chain fatty acids in plasma, fibroblasts, and amniocytes.
Zellweger syndrome is the most common peroxisomal disorder that presents in early infants with an incidence of 1 in 50,000 to 100,000 live births. The overall incidence of peroxisomal disorders is about 1 in 5 to 10,000 live births.
An 8-year-old female presents to the clinic with her father for concerns of erythematous pruritic papules that appeared suddenly on the dorsal surface of her bilateral hands. The patient’s father says he has been applying emollients and antifungal cream to the lesions without improvement. Differential diagnosis of actinic prurigo (AP) and polymorphous light eruption (PMLE) is made. Which of the following genetic factors can help in reaching the final diagnosis?
A. HLA-DR4
B. BRCA1
C. APOE e4
D. MTHFR
Both actinic prurigo (AP) and polymorphous light eruption (PMLE) can present with similar papular pruritic lesions and are thought to be due to a delayed hypersensitivity reaction induced by UV radiation.
There is a strong associated with actinic prurigo with the HLA-DR4 allele, specifically the DRB1*0407 subtype.
The human leukocyte antigen DR4 allele variant is present in 90% of cases of AP, but PMLE has no association with the DR4 allele.
AP has a strong genetic component while PMLE does not.
A 16-year-old boy presents with his mother to establish care after recently moving to the area. He was diagnosed with a rare genetic syndrome that affects skin, nails, and oral mucosa, but neither of them remembers its name. On examination, there is oral leukoplakia and poikiloderma. Short stature is also noticed. Given the most likely diagnosis, which of the following would be one of the first integumentary system signs to present?
A. Nail dystrophy
B. Reticulated hyperpigmentation
C. Hyperhidrosis
D. Telangiectasia
Dyskeratosis congenita (DKC), which is also known as Zinsser-Engman-Cole syndrome, is a genodermatosis. It is an uncommon syndrome classically associated with the triad of oral leukoplakia, nail dystrophy, and reticular hyperpigmentation.
Nail dystrophy is one of the first integumentary system signs of dyskeratosis congenital.
Usually, dystrophic nails appear between 5 to 13 years and show longitudinal fissures.
Reticulated hyperpigmentation and telangiectasias are common findings in dyskeratosis congenital, but these changes do not usually occur until after nail findings have been established.
XL inheritance and causes low telomere length, which can lead to bone marrow failure or cancers
A 44-year-old female presents to the office for a 3-day history of new floaters in her right eye and feeling ‘something is not right’ with her vision. She reports no history of ocular disease or trauma. Fundus examination reveals yellow-white choroidal lesions, one-quarter to one-half optic disc diameter, clustered around the optic nerve and the posterior pole, radiating towards the periphery, involving inferior and nasal peripapillary area, in a pattern similar to the gunshot spatter. What serum marker is most prevalent in this disease?
A. HLA-A29
B. HLA-DR2
C. HLA-B27
D. HLA-A17
Birdshot Uveitis has the strongest human class I MHC correlation with any disease, with 80-98% of patients being HLA-A29 positive, only 7% in the general population.
Diagnostic criteria involve more than 3 peripapillary birdshot lesions or cream-colored, irregular or elongated choroidal lesions with long axis radiating from the optic disc: less than 1+ anterior vitreous cells and less than 2+ vitreous haze in both eyes.
Supportive findings include positive serum marker HLA-A29 individuals.
HLA-DR2 is associated with pars planitis. HLA-B27 is associated with ankylosing spondylitis and other spondylarthropathies. HLA-A17 is not generally associated with ocular conditions.
A patient diagnosed with acute myeloid leukemia (AML) has a bone marrow biopsy taken with samples sent for sequencing. Clinical sequencing shows the patient harboring a heterozygous mutation in the gene NPM1, specifically, a four base pair insertion of the sequence TCTG. If the percentage of myeloblasts in the bone marrow by cytomorphology is 30%, what is the expected variant allele frequency (percentage of sequenced strands of DNA that are mutated) for the NPM1 mutation, assuming each AML blast harbors the mutation?
A. 10%
B. 15%
C. 30%
D. 60%
Since this mutation is heterozygous, every acute myeloid leukemia (AML) blast will have only one of the two homologous chromosomes coding for NPM1 mutated (chromosome 5). Thus, of all the chromosome 5’s residing in the patient’s AML cells, half are mutated. With 30% of the cells being AML blasts and half of those cells’ chromosomes being mutated, that means 15% of the patient’s DNA fragments sequenced should be mutated.
Since this mutation is heterozygous, every AML blast will have only one of the two homologous chromosomes coding for NPM1 mutated (chromosome 5). Thus, of all the chromosome 5’s residing in the patient’s AML cells, half are mutated. With 30% of the cells being AML blasts and half of those cells’ chromosomes being mutated, that means 15% of the patient’s DNA fragments sequenced should be mutated.
Since this mutation is heterozygous, every AML blast will have only one of the two homologous chromosomes coding for NPM1 mutated (chromosome 5). Thus, of all the chromosome 5’s residing in the patient’s AML cells, half are mutated. With 30% of the cells being AML blasts and half of those cells’ chromosomes being mutated, that means 15% of the patient’s DNA fragments sequenced should be mutated. If this mutation were homozygous, then this answer would be correct.
Since this mutation is heterozygous, every AML blast will have only one of the two homologous chromosomes coding for NPM1 mutated (chromosome 5). Thus, of all the chromosome 5’s residing in the patient’s AML cells, half are mutated. With 30% of the cells being AML blasts and half of those cells’ chromosomes being mutated, that means 15% of the patient’s DNA fragments sequenced should be mutated.
A 16-year-old girl is brought to the clinic with complaints of primary amenorrhea. There is no history of deepening of voice and appearance of facial hairs. Thelarche was attained at age 11. Pubic and axillary hairs are Tanner stage 5. External genitalia appears to be normal. The hymen is intact. Lab investigations show serum LH 6 IU/L (reference range: 24-105 IU/L), serum FSH 8 IU/L (reference range: 4-25 IU/L), serum testosterone 1.5 nmol/L (reference range: 0.52–2.4 nmol/L), and 7 beta-estradiol 250 pmol/L (reference range: 74-1468 pmol/L). USG abdomen shows the absence of a uterus, normal ovaries, and horseshoe kidney on the right side. Karyotype analysis reveals 46 XX phenotype. Based on the inheritance pattern of this condition, what is the most likely probability of the next child having this disease?
A. 25%
B. 50%
C. 100%
D. It cannot be calculated
The etiology of MRKH (Mayer-Rokitansky-Kuster-Hauser) syndrome is previously thought to be sporadic, but familial cases support genetic etiology. Although the precise gene has not yet been identified. MRKH syndrome is appeared to be transmitted in an autosomal dominant fashion.
MRKH syndrome causes the uterus or vagina to be underdeveloped or absent, although external genitalia and lower vagina are normal.
Affected women don’t have menstrual periods due to an absent uterus.
Women with MRKH syndrome have a female chromosome pattern and normally functioning ovaries.
A 1-month-old girl with a history of limb deformities, hearing loss, and developmental delay presents with a rash. The physical examination demonstrates striking unilateral erythematous scales with clear midline demarcation on the left side of her body, short stature, facial hemidysplasia, and right upper extremity hypoplasia. Chest x-ray shows spinal scoliosis. Given the likely diagnosis, which complication is most associated with this patient’s condition early in infancy?
A. Arachnodactyly
B. Megacolon
C. Stippled calcification of the epiphysis
D. Omphalocele
CHILD (congenital hemidysplasia with ichthyosiform erythroderma and limb defects) syndrome is inherited in an X-linked dominant pattern, and therefore a preponderance of those affected are female (19:1 female:male).
The clinical manifestations are ipsilateral hypoplasia of the limbs, facial hemidysplasia, and hypoplasia of various organ systems.
The classic epidermal findings in CHILD syndrome include striking unilateral erythematous scales with clear midline demarcation that tend to improve with age.
Stippled calcification of the epiphysis, a unique bone finding, can be found early in infancy and can help clinicians in making the diagnosis.
A 2-year-old boy is brought in by his parents as they have noticed he is not achieving developmental milestones. The mother is concerned as her son is not walking yet and is not able to speak. She describes jerky movements of the limbs with no loss of consciousness. On examination, there is a significant cognitive delay, spasticity, and hyperreflexia, and ataxia. Noisy breathing is noted as well. An MRI reveals reduced white matter and remarkable pontine hyperintensity on T2. Based on the results, a demyelinating disorder is suspected and orders genetic testing. At what gene is a mutation expected in this patient?
A. GJC2
B. PLP1
C. L1CAM
D. SOX10
Pelizaeus-Merzbacher-like disease (PMLD) is a leukodystrophy that presents in a very similar manner to Pelizaeus-Merzbacher disease (PMD). It is an autosomal recessive disorder, unlike PMD, which is X-linked recessive.
PMLD is due to a mutation at the level of the GJC2 gene. Clinical features include spasticity, delayed milestones, cognitive and motor impairment, titubation, and others. Clinical features are almost identical to PMD.
Radiological findings include demyelination of most of the brain structures on MRI in both PMD and PMLD along with T2 hyperintensity. However, the involvement of the pontine area is more suggestive of PMLD.
PLP1 mutation leads to PMD, L1CAM mutation leads to metachromatic leukodystrophy, and SOX10 mutation leads to Waardenburg syndrome.
A 34-year-old man presents to the office after confirmatory testing revealing HIV positivity. A therapeutic regimen consisting of multiple antiretroviral therapies is presented to him, one of which is the drug known as abacavir. Testing for the presence of HLA B 5701 is performed, and the result is positive. Which of the following types of hypersensitivity is most likely to occur if this patient receives this treatment?
A. Type I
B. Type II
C. Type III
D. Type IV
Abacavir is a drug used in antiretroviral HIV regimens as an adjunct to treatment. Abacavir mediated hypersensitivity is a type IV hypersensitivity reaction in susceptible patients. The presence of the HLA B 5701 allele is a contraindication to abacavir treatment.
Symptoms present within 2 weeks of drug initiation and occur as a result of a T-cell mediated cytokine reaction. Symptoms most commonly include rash, fever, fatigue, and GI disturbances.
Hypersensitivity to abacavir occurs in 5-8% of patients who receive treatment and has therefore warranted genetic screening prior to initiation.
Abacavir is a nucleoside reverse transcriptase inhibitor (NRTI) and exerts its action chain termination following incorporation into viral DNA.
A 29-year-old man presents to the clinic due to an abnormal elevation of blood sugar. One month ago, the patient started to notice increased urination associated with thirst. He visited his primary healthcare provider and underwent blood investigations to figure out the underlying cause. His fasting blood glucose level was 150 mg/dL and 160 mg/dL on two separate settings (reference range <100 mg/dL). The patient mentions that his young brother has diabetes mellitus. Renal ultrasound reveals numerous small cysts. Which of the following is the next best step in managing this patient?
A. Renal biopsy
B. Abdominal computed tomography
C. HNF1B genetic testing
D. Echocardiography
Autosomal dominant tubulointerstitial kidney disease (ADTKD) is a group of inherited kidney disorders caused by different gene mutations that share the same histological findings. The clinical manifestations are widely variable and gene-dependent. To date, many different gene mutations that cause ADTKD have been detected: uromodulin (UMOD), renin (REN), mucin-1 (MUC1), hepatocyte nuclear factor 1-beta (HNF1B), and recently SEC. 61A1 mutation was identified as a novel cause of this disease.
In ADTKD-HNF1B, the HNF1B gene is essential in early embryonic development. It is also known as transcription factor-2 (TCF2). This gene is located on chromosome 17q12, which codes for protein hepatocyte nuclear factor 1 homeobox B. Since HNF1B is expressed in the body’s variable tissues like the kidney, liver, pancreas, and genitourinary tract, this may explain the variable clinical presentations of the HNF1B-related disorders. It was found that loss of HNF1b could induce epithelial-mesenchymal transition. This type of transition will ultimately lead to kidney fibrosis by up-regulation of the expression of transforming growth factor-beta ligands in renal epithelial cells.
ADTKD-HNF1B is caused by a mutation in the TCF2 gene. The homeodomain-containing transcription factors hepatocyte nuclear factor 1 alpha and 1 beta are expressed both in the kidney and the pancreas. It has been identified that mutations in this gene may cause the maturity-onset diabetes of youth (MODY), but further studies explicate that mutations in this gene may also lead to renal manifestations. The varied clinical manifestations and the fact that these clinical manifestations are not present consistently in all affected family members made diagnosing this condition difficult. The non-renal manifestations may include maturity-onset diabetes in youth, abnormal liver function tests of unclear etiology, genitourinary tract malformations, hyperuricemia with gout in adolescence, and hypomagnesemia may occur in some individuals. The renal manifestations are common in adulthood and often involve numerous renal cysts, congenital solitary kidney, congenital anomalies of the kidney and urinary tract system, renal dysplasia, and hypoplastic kidneys.
For ADTKD-HNF1B, there is no specific therapy for this disease. The treatment is mainly supportive of chronic kidney disease. Allopurinol or febuxostat should be considered for patients with early-onset gout to prevent tophus development and urate accumulation. Screening for abnormal liver function tests, hyperglycemia, hypomagnesemia, and hyperuricemia is an essential part of the management to prevent further complications. To evaluate the renal morphologic abnormalities, renal ultrasound should be performed.
A 39-year-old male presents to the genetics clinic for evaluation of multisystemic malignancies. His father passed away from complications relating to an adrenal pheochromocytoma and his paternal uncle was diagnosed with pancreatic insufficiency, a pancreatic malignancy, and an unspecified brain tumor. A CT of the patient’s kidneys show unilateral large cysts. Which of the following brain tumors is the patient most likely predisposed to get in his lifetime?
A. Hemangioblastoma
B. Meningioma
C. Choroid plexus papilloma
D. Glioblastoma multiforme
The CT image demonstrates a large right renal mass and a smaller left renal mass. Based on the patient’s family history and his own personal history of renal malignancy, the patient is likely the carrier for the autosomal dominant mutation of the VHL tumor suppressor gene(Von Hippel Lindau Syndrome).
Von Hippel Lindau Syndrome is characterized by renal cell carcinomas typically of the clear cell subtype. There is a high incidence of renal cancer in VHL patients (up to a 70% lifetime risk). Renal cysts are also commonly seen and renal angiomyolipomas are also frequently incidentally discovered on imaging.
Other tumors include pancreatic neuroendocrine tumors, pheochromocytomas, and pancreatic adenocarcinoma. More often the pancreas is affected by cystic replacement (pancreatic cystosis) or serous cystic neoplasms.
Von Hippel Lindau Syndrome patients often also acquire CNS hemangioblastomas, most commonly within the cerebellum. Hemangioblastomas of the spinal cord and brainstem can also occur, although less commonly noted. Choroid plexus papilloma, while also being associated with VHL patients, are less common than hemangioblastomas.
A 13-year-old right-handed Japanese boy presented to the emergency department after being found in his sister’s house with generalized tonic-clonic seizures. The patient was treated with lorazepam and became post-ictal. He was given valproic acid. He improved and was alert and oriented, following commands and moving all extremities symmetrically, however, he had a finger to nose dysmetria bilaterally, ataxic gait, and a positive Romberg test. When the older sister provided collateral information, she stated that seizures, difficulties walking, memory problems, and odd “dance-like movements” ran in the family. The paternal father, grandfather, and great-aunts have the condition. Genetic testing was pending. Which of the following is the most appropriate regarding mortality in this condition?
A. Life expectancy is about 20-30 years
B. Recurrent uncontrolled seizures lead to mortality in most cases
C. Aspiration pneumonia is one of the most feared complications
D. The lower number of CAG repeats leads to higher mortality and will present with status epilepticus
Dentatorubral pallidoluysian atrophy (DRPLA) is classified as a microsatellite repeat disorder, which shows genetic anticipation with earlier onset and worse disease severity as it passes to the next generation. Studies show that functional impairment is an indirect measure of disease severity, given that it is inversely related to the CAG repeat length.
DRPLA patients require structured physical and occupational therapy to preserve, motor mobility, and functional activities of daily living as much as possible. Structured and individualized educational programs for affected children are other important considerations.
An extensive neuropsychological and psychiatric assessment is instrumental in evaluating progressive memory and mood disorders. Managing them early will preserve more cognitive domains and increase the quality of life.
Life expectancy ranges from 8–16 years from symptom onset. The length of the CAG repeat expansion in ATN1 is positively correlated with age of onset, clinical phenotype, and life expectancy, with longer repeats being linked to earlier onset, severe disease phenotype, and a shorter lifespan. The most feared complications are status epilepticus, aspiration pneumonia, and recurrent seizures, which are all linked with increased mortality.